Location via proxy:   [ UP ]  
[Report a bug]   [Manage cookies]                
0% found this document useful (0 votes)
4 views

lecture_notes math 1

The document discusses the concepts of angles, radians, and trigonometric functions, defining angles in standard position and introducing the unit circle for measuring angles in radians. It explains the sine and cosine functions based on the coordinates of points on the unit circle and extends to define additional trigonometric functions. Additionally, it covers the concept of limits in functions, providing definitions, examples, and rules for evaluating limits, including one-sided limits.

Uploaded by

shh25ytk6t
Copyright
© © All Rights Reserved
Available Formats
Download as PDF, TXT or read online on Scribd
0% found this document useful (0 votes)
4 views

lecture_notes math 1

The document discusses the concepts of angles, radians, and trigonometric functions, defining angles in standard position and introducing the unit circle for measuring angles in radians. It explains the sine and cosine functions based on the coordinates of points on the unit circle and extends to define additional trigonometric functions. Additionally, it covers the concept of limits in functions, providing definitions, examples, and rules for evaluating limits, including one-sided limits.

Uploaded by

shh25ytk6t
Copyright
© © All Rights Reserved
Available Formats
Download as PDF, TXT or read online on Scribd
You are on page 1/ 210

Trigonometric Functions

Angles

idea of Definition: An angle is formed by rotating a ray about its endpoint.


An angle is said to be in standard position if its vertex is at the origin and its initial
side is along the positive 𝑥𝑥-axis.
Note An angle in standard position is uniquely determined by the direction and
magnitude of rotation. So we can use numbers to represent angles.

• The initial position of the ray is called the initial side.


• The endpoint of the ray is called the vertex.
• The final position is called the terminal side.
The direction of rotation may be counterclockwise or clockwise which will be
considered to be positive or negative respectively.
• Magnitudes of rotation are traditionally measured in degrees where one
revolution is defined to be 360 degrees, written 360° .
Figures (a), (b) and (c) show three angles in standard position: Although the
angles have the same terminal sides, their measures are different.

(a) (b) (c)


Radians

Another unit for measuring angles is the radian. To define radian, we consider
unit circles.
Terminology: A circle with radius 1 is called a unit circle. The circle with radius
1 and center at the origin is called the unit circle.
Definition The angle deterrnined by an arc of length 1 along the circurn-ference
of a unit circle is said to be of measure one radian.

Since the circurnference ofa unit circle has length 2𝜋𝜋, there are 2𝜋𝜋 radians in one
revolution. Therefore, we have 360° = 2𝜋𝜋 radians. The conversion between
degrees and radians is given by

Exercise
1. Convert the following degree measures to radians:

a)270° b) 315°

2. Convert the following radian measures to degrees:

𝜋𝜋 5𝜋𝜋
a) b)
6 2
Trigonometric Functions

Notation Consider an angle ϴ in standard position. Let P be the point of


intersection of the terminal side and the unit circle.

We define sin 𝜃𝜃 = y-coordinate of P, cos 𝜃𝜃 = x-coordinate of P.


Remark : Instead of considering the unit circle, we can also use circle of
𝑎𝑎 𝑏𝑏
radius r ( centered at the origin) and define cos 𝜃𝜃 = and sin 𝜃𝜃 = ,
𝑟𝑟 𝑟𝑟
where a and b are the x- and y-coordinates of P respectively. It is easily
seen (using sirnilar triangles) that these ratios are independent of the
choice of r.
Definition The rules given sin 𝜃𝜃 = y-coordinate of P and cos 𝜃𝜃 = x-coordinate of
P define two functions from ℝ into ℝ. called the sine and cosine functions
respectively.
Using the sine and cosine functions, we define four more trigonometric functions,
called the tangent ( de-noted by tan), cotangent (denoted by cot), secant (denoted
by sec) and cosecant (denoted by csc) functions as follows:
D�...fi ıırHO('I ; Le+ no k-- o re.o.l nv-rıher
A 1\JfıC..+ıa, -f i� pc:riod.c of- T pCJ-iod if flx.+-r)==--f lx) fa-. ali x. svc.h -+hCt+
x ond X+T are il'\ d;:nıaı'" of- f.
· The sn-ıall-es+ T ıs called � c::rq-Qt\ ?e lad o"d.
r
ckııo+ed. b.j P

8X flx.).=: x- QxO sl--o..J..J � +' )� periodıc or- "o,t.


j
� m f- 7t ard. a 6 114
• v:)< -tavıJOLJ1 U)ırli �
Qo\�mLJ= DaO +r ol�

flx+T):; X+T-[lx-ı--,!]

ı t ıt· � = X + T-((_{ xl] -t-1)


:= x-OxO
:::-.f'lx) TE zı-t: p:=l

sin0- ��

cos9-== ..!..
'
.1
1t,n$= sı� co+-e- =-� s �c. ı9-.: -1-
COJ/9-
ese�-=-_j_
�$ Sı� Jfr-'$,

- co� l& -t-Sm 'l.Q.--::_L


Sil'\ (A + 8) = Stf'\A '-D.St +- S.N'\ B . l.O..S A
=
S\--ı (A-&) S:ı'r-.A. c...a�8 - .SıııB>, '-'='�.A
CD� (A+I?.) =
��A--e,o�g - SrnA..s,1rı! - -to.f\ lA +" t J=- - -t tcn.S
fıaıı A

- cc.1 (A-S)-:; c.osA-.c.os'S + s:&"\A �;,,&


strı2A � l- �ıııA. A c..o� I +- filt\ A +oıı g
ea.s.2.A = co� 1A-s.ın ıA
ws2A-= .1-.1.Sı'/"l .1. A
=
eos.2.A Uos2 A-1
sioA.c.o�i!,: [ "± "'"'
lA+&)+ �ın (A-Bj

Co.}A ı:.o.ı e, r i [c,:,.ı C A + g ) +- c.c-ı CA- S]


Sif\A . .s,ıı& =i [-co�lA-t 8)+-ceı� CA-t)J
π
−1 ≤ x ≤ 1 , arccos x = 2 − arcsin x
y = cot −1 x = arc cot x ⇔ x = Cot y

⇔ x = cot y, 0 < y < π


1 Limits of Functions
Definition 1 An informal definition of limit
If f (x) is defined for all x near a, except possibly at a itself, and if we can
ensure that f (x) is as close as we want to L by taking x close enough to a,
but not equal to a, we say that the function f approaches the limit L as x
approaches a, and we write

lim f (x) = L.
x!a

The following alternative notation is sometimes used

f (x) ! L as x ! a;

(read ”f (x) approaches L as x approaches a” or ”f (x) goes to L as x goes to


a”.)

Example 2 substituting numbers to guess a limit. What (if anything) is

x2 2x
lim ?
x!2 x2 4
2
We first try to substitute x = 2, but this leads to f (2) = 222 2.2 0
4 = 0 which does
not exist. Next we try to substitute values of x close but not equal to 2. Table 1
suggests that f (x) approaches 0.5.

x f (x) x f (x)
3.000000 0.600000 1.000000 0.333333
2.500000 0.555556 1.500000 0.428571
2.100000 0.512195 1.900000 0.487179
2.010000 0.501247 1.990000 0.498746
2.001000 0.500125 1.999000 0.499874

Table 1: Finding limits by substituting values of x close to a.

Example 3 If f (x) = x + 3 then

lim f (x) = 7,
x!4

is true, because if you substitute numbers x close to 4 in f (x) = x + 3 the result


will be close to 7.

Example 4 (a) If f is the identity function, f (x) = x, then for any value
of x0 ,
lim f (x) = lim x = x0 ,
x!x0 x!x0

1
(b) If f is the constant function, f (x) = k (function with the constant value
k), then for any value of x0 ,

lim f (x) = lim k = k.


x!x0 x!x0

For instances of each of these rules we have

lim x = 3
x!3

and
lim 4 = lim 4 = 4.
x! 7 x!2

Example 5 Evaluate:
!
***
x2 + x 2 1 1
(a) lim
x! 2 x2 + 5x + 6 (b) lim x a

F
x!a x a
= 3

Solution Each of these limits involves a fraction whose numerator and de-
nominator are both 0 at the point where the limit is taken.
(a)

x2 + x 2 (x + 2)(x 1) x 1 2 1
lim 2
= lim = lim = = 3
x! 2 x + 5x + 6 x! 2 (x + 2)(x + 3) x! 2 x + 3 2+3

(b)
1 1 a x
lim x a = lim ax = lim 1 = 1
.
x!a x a x!a x a x!a ax a2
Remark 6 Always be aware that the existence of limx!a f (x) does not require
that f (a) exist and does not depend on f (a) even if f (a) does exist. It depends
only on the values of f (x) for x near but not equal to a.

1.1 One-Sided Limits


Definition 7 Informal definition of left and right limits
Let a 2 R and let f be a function such that f (x) is defined for x su↵ciently
close to and greater than a. Suppose L is a real number satisfying f (x) is
arbitrarily close to L if x is su↵ciently close to and greater than a. Then we
say that L is the right-side limit of f at a and we write limx!a+ f (x) = L.
If a function f is defined on the left-side of a, we can consider its left-side
limit. The notation limx!a f (x) = L means that f (x) is arbitrarily close to L
if x is su↵ciently close to and less than a.

2

x + 1, x 1
Example 8 g(x) =
x 1, x<1

lim g(x) = lim x 1=0


x!1 x!1

lim g(x) = lim x + 1 = 2


x!1+ x!1+

Theorem 9 A function f (x) has limit L at x = a if and only if it has both left
and right limits there and these one-sided limits are both equal to L:

lim f (x) = L , lim f (x) = lim+ f (x) = L.


x!a x!a x!a

Example 10 If f (x) = bxc, find: limx!4+ f (x), limx!4 f (x) and limx!4 f (x).

Solution
Observe that f (x) = bxc = 4 if x > 4 and f (x) = bxc = 3 if x < 4.

Therefore, limx!4+ bxc = 4 and limx!4 bxc = 3.


Since limx!4+ f (x) 6= limx!4 f (x), the limit limx!4 f (x) does not exist.

|x|
Example 11 Show that limx!0 does not exist.
x

Solution
|x| x
limx!0 = limx!0 = limx!0 ( 1) = 1
x x
and
|x| x
limx!0+ = limx!0+ = limx!0+ 1 = 1.
x x
|x| |x| |x|
Since limx!0 6= limx!0+ , the limit limx!0 does not exist.
x x x
⇢ 2
y + 5, y < 2
Example 12 Given the function, g(y) =
1 3y, y 2

Compute the following limits.

(a) limy!6 g(y)


(b) limy! 2 g(y)

Solution

(a) In this case there really isnt a whole lot to do. In doing limits recall that
we must always look at whats happening on both sides of the point in question
as we move in towards it. In this case y = 6 is completely inside the second
interval for the function and so there are values of y on both sides of y = 6

3
that are also inside this interval. This means that we can just use the fact to
evaluate this limit.
lim g(y) = lim (1 3y) = 17.
y!6 y!6

(b) In this case the point that we want to take the limit for is the cuto↵ point
for the two intervals. In other words, we cant just plug y = 2 into the second
portion because this interval does not contain values of y to the left of y = 2
and we need to know what is happening on both sides of the point.

lim g(y) = lim (y 2 + 5) = 9


y! 2 y! 2

and
lim g(y) = lim + (1 3y) = 7.
y! 2+ y! 2

So, in this case we can see that,

lim g(y) = 9 6= 7 = lim + g(y)


y! 2 y! 2

and so since the two one sided limits aren’t the same

lim g(y)
y! 2

doesnt exist.

Theorem 13 Rules for Limits of Functions at a Point


If L, M , a and k are real numbers and limx!a f (x) = L and limx!a g(x) =
M, then
1. limx!a (f (x) + g(x)) = L + M

2. limx!a (f (x) g(x)) = L M


3. limx!a (k.f (x)) = k.L
4. limx!a (f (x).g(x)) = L.M
f (x) L
5. limx!a = , M 6= 0
g(x) M
n
6. limx!a [f (x)] = Ln , n is a positive integer
p p 1
7. limx!a n f (x) = n L = L n , n is a positive integer. (If n is even, we
assume that limx!a f (x) = L > 0.)

8. If f (x)  g(x) on an interval containing a in its interior, then L  M.

4
Example 14 Evaluate
x3 2x2 5
lim .
x!3 8 4x

Solution
x3 2x2 5 limx!3 x3 2x2 5
lim =
x!3 8 4x limx!3 8 4x
limx!3 x3 2 limx!3 x2 limx!3 5
=
limx!3 8 4 limx!3 x
limx!3 x. limx!3 x. limx!3 x 2 limx!3 x. limx!3 x limx!3 5
=
limx!3 8 4 limx!3 x
3.3.3 2.3.3 5
=
8 4.3
4
=
4
= 1

Theorem 15 Limits of Polynomials and Rational Functions


1. If P (x) is a polynomial and a is any real number, then >
-
pol!nomlar sürekl! olduğu !ç!n

L!m!tler! da!ma vardır.

lim P (x) = P (a).


x!a

2. If P (x) and Q(x) are polynomials and Q(a) 6= 0, then

P (x) P (a)
lim = .
x!a Q(x) Q(a)

x2 4
Example 16 Evaluate lim .
x!4 x + 2

Solution

x2 4 16 4
limx!4 = = 2.
x+2 4+2
Isandw!ch theorem)
Theorem 17 The Squeeze Theorem ( P!nch!ng theorem)
Suppose that
f (x)  h(x)  g(x)
for all x in some open interval containing c and that

lim f (x) = lim g(x).


x!c x!c

Then
lim f (x) = lim h(x) = lim g(x).
x!c x!c x!c

5
The theorem is useful when you want to know the limit of h, and when
you can sandwich it between two functions f and g whose limits are easier to
compute. There is an important di↵erence: in the Squeeze Theorem you don’t
have to assume that the limit of h exists. The inequalities f  h  g combined
with the circumstance that f and g have the same limit are enough to guarantee
that the limit of h exists.
The following figure illustrates what is happening in this theorem.

Figure 1

The Squeeze theorem is also known as the Sandwich Theorem and the Pinch-
ing Theorem.

Example 18 Show that if limx!a | f (x) |= 0, then limx!a f (x) = 0. -

IF(x)) = f(x) IF(X))

l!m IF(x)) = 0 and


Solution Since | f (x) | f (x) | f (x) | and | f (x) | and | f (x) | both
X+ a
have limit 0 as x approaches a, so does f (x) by the Squeeze Theorem.
l!m ( -1) (f(x)) = D
Example 19 An Important Limit X- a

sinx
limx!0 =1
x
öneml!
Proof.

6
Figure 2


First we consider right-side limit. Let x be small, positive (0 < x < ).
2
Consider the triangles 4OAB and 4OAC and the sector OAB shown in Figure
2.
Note that
1 sinx
area of 4OAB = .1.1.sinx =
2 2
1p x
area of sector OAB = . 1.x =
2 2
1 tanx
area of 4OAC = .1. | AC |=
2 2
Since 4OAB ✓ sectorOAB ✓ 4OAC, it follows that
sinx x tanx
< < .
2 2 2
sinx
Dividing each term by (which is positive), we get
2
x 1
1< < ,
sinx cosx
which, by taking reciprocal, yields
sinx
1> > cosx.
x
By the continuity of the cosine function (at 0), we have

lim cosx = cos0 = 1.


x!0

Letting x ! 0+ with the Sandwich Theorem (which is also valid for limits at a
point and one-sided limits), we get
sinx
lim+ = 1.
x!0 x
sinx sin( x) sinx
Since is an even function, that is, = for all x 6= 0 it follows
x x x
sinx sinx
that limx!0 = 1. Therefore, we have limx!0 = 1.
x x

7
Remark 20 The result means that if x is small, then sinx is approximately
equal to x.

Example 21 Evaluate the following limits.


tan x
(a) limx!0 =?
x
tan x sin x 1
lim = lim . =1
x!0 x x!0 x cos x
1 x 1
(b) limx!0 2
. sin2 =?
x 2
1 2 x sin x2 2 1 2 1
lim . sin = lim ( x ) .( ) =
x!0 x2 2 x!0 2 4
2
p
sin 2x
(c) limx!0+ p =?
sin 2x
p
sin 2x
p
sin 2x
p
2x
- m
lim+ p = lim+ ( p .p )
x!0 sin 2x x!0 2x sin 2x
p r
sin 2x 2x
= lim+ p . lim+
x!0 2x x!0 sin 2x
-
= 1.1 = 1 1

. -
sin(2x)
(d) limx!0 =? =

sin(3x)
sin(2x) sin(2x) 2x 3x 2
lim = lim . . =
x!0 sin(3x) x!0 2x 3x sin(3x) 3

Definition 22 Let f be a given function. We say that f is bounded if there is


a positive number M such that | f (x) | M for any x in domain.
-
bu 2 koşul sağlanmalı
Theorem 23 If f is a bounded function and limx!a g(x) = 0 then

lim f (x).g(x) = 0.
x!a

#!lk
✓ ◆
1
Example 24 limx!0 x. sin
x
=? e!m teorem as
kullanamam

1 o yüzden
Since sin is a bounded function and limx!0 x = 0 then we have 0 olduğunu
x L!m X =

✓ ◆ X +0 kullanarak
1
lim x. sin = 0. o olduğunu
x!0 x bulduk.

8
(2 + h)2 4
Example 25 limh!0 =?
h

(2 + h)2 4 4 + 4h + h2 S
/ 4
lim = lim
h!0 h h!0 h
4h + h2
= lim
h!0 h
= lim 4 + h
h!0
= 4

-
p
x2 6x + 9
Example 26 limx!3+ =?
x 3
p p
x2 6x + 9 (x 3)2
lim = lim+
x!3+ x 3 x!3 x 3
|x 3|
= lim
x!3+ x 3
x 3
= lim
x!3+ x 3
= lim+ 1
x!3
= 1
sin x tan x >

s!nx-
-

Example 27 limx!0 =?
x3
1
sin x tan x sin x(1 )
lim = lim cos x
x!0 x3 x!0 x.x2
1
sin x 1 cos x
= lim .
x!0 x x2
1
sin x 1
= lim . lim cos x
x!0 x x!0 x 2
-
1 1
1
= lim cos x
x!0 x2 - - s!n2x
cos x 1 cos x + 1
= lim .
x!0 cos x.x2 cos x + 1
sin x. sin x 1
= lim .
x!0 x.x cos x(cos x + 1)
1
=
2
cos(2x) 1
Example 28 limx!2⇡ =?
sin2 (2x) > 1 cos"(2x)
-
-

h
9
cos(2x) 1 cos(2x) 1
lim = lim
x!2⇡ sin2 (2x) x!2⇡ 1 cos2 (2x)
cos(2x) 1
= lim
x!2⇡ (1 cos(2x))(1 + cos(2x))
1
= lim
x!2⇡ 1 + cos(2x)
1
=
1 + cos(4⇡)
1
=
2
s
| 4 x2 |
Example 29 limx! 2 =?
|2+x|
s s
| 4 x2 | |2 x || 2 + x |
lim = lim
x! 2 |2+x| x! 2 |2+x|
p
= lim |2 x|
x! 2
25 . 10 . 23
= 2
1 6
N Example 30 limx!3 ( ) =?
x 3 x2 9
1 6 x+3 6
lim ( ) = lim
x!3 x 3 x2 9 x!3 x2 9
x 3
= lim
x!3 x2 9
x 3
= lim
x!3 (x 3)(x + 3)
1
= lim
x!3 x + 3
1
=
6
p p
5x 5 x
Example 31 limx!5 =?
x 5

10
p p p p p p
5x 5 x ( 5x 5 x)( 5x + 5 x)
lim = lim p p
x!5 x 5 x!5 (x 5)( 5x + 5 x)
5x2 25x
= lim p p
x!5 (x 5)( 5x + 5 x)
5x(x 5)
= lim p p
x!5 (x 5)( 5x + 5 x)
5x
= lim p p
x!5 5x + 5 x
25
= p p
5 5+5 5
p
5
=
2
p
x+4 2
z Example 32 limx!0 =?
x
p p p
x+4 2 ( x + 4 2)( x + 4 + 2)
lim = lim p
x!0 x x!0 x( x + 4 + 2)
x+4 4
= lim p 1)
x!0 x( x + 4 + 2) katsayısı
(x'!m

yapma!n
1
=
4 >
-

2
x + 2x 3 Horner Methodu
~ Example 33 limx!1 =?
2x3 + x2 2x + 1 pol!nomda bölme
2x3 + x2 1/

(2x2
2x +

-
2 laraştır)
x + 2x 3 (x + 3)(x 1)
-

2x 2xh +3x +1
-
lim = lim
- - x!1 2x3 + x2 2x + 1 x!1 (x
1)(x + 1)(2x + 1)
3x2 -
2x 1
-

-
=
6 2
x3 y x2 y 2
Example 34 limx!y =?
x4 y4
x3 y x2 y 2 x2 y(x y)
lim = lim
x!y x4 y4 x!y (x2 y 2 )(x2 + y 2 )
x2 y(x y)
= lim

(x
x!y (x y)(x + y)(x2 + y 2 )
2
y y
=
(y + y)(y 2 + y 2 )
y3

2
=
4y 3
1
+ y2 =
4

: 11

=
"()
Mosco
1 cos x
v
Example 35 limx!0 =? =

sin x
1 cos x 1 (1 2 sin2 x2 )
lim = lim
x!0 sin x x!0 2 sin x2 cos x2
2 sin2 x2
l!m
s!n =
=

= lim
x!0 2 sin x cos x
2 2
sin x2
= lim
x!0 cos x
2
0
=
1
= 0
3* ,
0
x= 0 cutoff not
1x1 =

Example 36 Evaluate the following limits.


(a) limx!4 (x bxc) =?
sgn(x)[X = 0 cuto

lim (x
x!4
bxc) = lim (x
x!4
3) = 1
ma - galer!de

lim (x bxc) = lim+ (x 4) = 0


x!4+ x!4

Hence, the limit does not exist.


3x
(b) limx!0 bxc =?

3x 3x
lim = lim =0
x!0 bxc x!0 1
bunları çöz
Example 37 Evaluate the following limits.
-
1. limt!5
t3 6t2 + 25
(answ. 15) st-t +
-
-
t 5
2( 3 + h)2 18
2. limh!0
p
h
(answ. 12) +
29-6hth)
t 3t + 4 5 =
+ + 5

-
O
-

3. limt!4 (answ. )
4 t 8
✓ ◆ 25 -
5 - 5 =
15/
2 1
4. limx!0 x cos (answ. 0) 24 12
x
-

e!m xcos
O

1 cosx 1
5. limx!0 (answ. )
x2 2
6. limx!3 bsgn(x2 9) + sgn(x 4)c (answ. limit doesn’t exist)
|x|+| x|
7. limx!0 (answ. 0)
sgn(x)

12
i
8. limx!3 bx bxcc (answ. 0)
9. limx!3 bxc (answ. limit doesn’t exist)
|x 2|
10. limx!1 (answ. 1)
sgn(x 1)2

2 Limits of Sequences
Definition 38 A sequence is a function whose domain is Z+ (the set of all
positive integers). A sequence of real numbers is a sequence whose codomain is
R.

Let f : Z+ ! R be a sequence. For each positive integer n, the value f (n)


is called the nth term of the sequence and is usually denoted by a small letter
together with n in the subscript, for example an . The sequence is also denoted
by (an ). 9)
aler!de
ar

Sometimes the numbers in a sequence approach


✓ a single value
◆ as the index M
1 1 1 1
n increases. This happens in the sequence 1, , , , ..., , ... whose terms
>
-

(1)l!m -
✓2 3 4 n ◆
1 2 3 1
approach 0 as n gets large, and in the sequence 0, , , , ..., 1 , ... whose (2) 1-1 approaches
>
-
to
2 3 4p p pn
p
terms approach 1. On the other hand, sequences like 1, 2, 3, ..., n, ...
S

have terms that get larger than any number as n increases.

Definition 39 A sequence (an ) is said to be convergent if there exists a real


number L such that (an ) is arbitrarily close to L if n is su↵ciently large.

In the definition of convergent, it is clear that if L exists, then it is unique.


We say that L is the limit of (an ) and we write limn!1 an = L.

Theorem 40 Rules for Limits of Sequences


If (an ) and (bn ) converge, then
1. limn!1 k = k (where k is a constant),
2. limn!1 (an + bn ) = limn!1 an + limn!1 bn ,
3. limn!1 (an bn ) = limn!1 an limn!1 bn ,
4. limn!1 (an .bn ) = (limn!1 an ).(limn!1 bn ),
5. limn!1 (k.an ) = k. limn!1 an (where k is a constant),
✓ ◆
an limn!1 an
6. limn!1 = provided that limn!1 bn 6= 0,
bn limn!1 bn
7. If an  bn , then limn!1 an  limn!1 bn ,
8. If an  bn  cn and limn!1 an = L = limn!1 cn , then limn!1 bn = L. Vers!on of Squeeze
>
-

Isea .

theorem)

13
✓ ◆
1
s
Example 41 Find limn!1 4 + n , if it exists.
2
Explanation
1
The sequence under consideration is given by an = 4 + n . The question
2
asks for the following:
1. Does the limit of (an ) exist or not (or equivalently, is the sequence con-
vergent)?
2. If the answer to (1) is affirmative, find the limit.
Solution
✓ ◆
1 1
lim 4+ = lim 4 + lim =4+0=4
n!1 2n n!1 n!1 2n

2n3 3n2 + n
~ Example 42 Find limn!1 , if it exists.
6n3

Solution

✓ ◆
2n3 3n2 + n 1 1 1 1 1 1 1 1
lim = lim + = lim . lim + . lim
n!1 6n3 n!1 3 2n 6n2 n!1 3 2 n!1 n 6 n!1 n2
1 1 1 1
= .0 + .0 = .
3 2 6 3
~ Example 43 Find limn!1 (1 + 2n), if it exists.

Solution
Limit does not exist. This is because we cant find any real number L satis-
fying the condition that 2n + 1 is close to L if n is large.
Example 44 Calculate the limits of the sequences.
✓ 2 ◆ ⇣ ⌘ p
~ (a)
2n n 1 ~(b) cosn ~ (c) n2 + 2n n
5n2 + n 3 n

Solution

(a) We divide the numerator and denominator of the expression for an by


the highest power of n in the denominator, that is, by n2 :
2n2 n 1 2 (1/n) (1/n2 ) 2 0 0 2
lim = lim = = ,
n!1 5n2 + n 3 n!1 5 + (1/n) 2
(3/n ) 5+0+0 5
1 1
since limn!1 = 0 and limn!1 2 = 0. The sequence converges and its limit
n n
2
is .
5

14
(b) Since | cosx | 1 for every n, we have
1 cosn 1
 
n n n
for n 1.
1 1
Now, limn!1 = 0 and limn!1 = 0, Therefore, by the sequence
n n
version of the Squeeze Theorem, limn!1 (cosn/n) = 0. The given sequence
converges to 0.

(c) For this sequence we multiply the numerator and the denominator (which
is 1) by the conjugate of the expression in the numerator:
⇣p ⌘ p p
n2 + 2n n n2 + 2n + n
lim n2 + 2n n = lim p
n!1 n!1 n2 + 2n + n
2n
= lim p
n!1 2
n + 2n + n
2
= lim p = 1.
n!1 1 + (2/n) + 1

The sequence converges to 1.

Example 45 For each of the following, find the limit if it exists.


1
1. limn!1 p
n
✓ ◆
5
2. limn!1 7
n2
3n2 4000
3. limn!1
2n2 + 10000
n2 12345
4. limn!1
n+1
5n2 + 4
5. limn!1
2n3 + 3
✓ ◆
1
6. limn!1 + ( 1)n
n

3 Limits at Infinity and Infinite Limits


3.1 Limits at Infinity
Definition 46 Limits at Infinity If the function f is defined on an interval
(a, 1) and if we can ensure that f (x) is as close as we want to the number L

15
by taking x large enough, then we say that f (x) approaches the limit L as x
approaches infinity, and we write

lim f (x) = L.
x!1

If f is defined on an interval ( 1, b) and if we can ensure that f (x) is


as close as we want to the number M by taking x negative and large enough
in absolute value, then we say that f (x) approaches the limit as x approaches
negative infinity, and we write

lim f (x) = M.
x! 1

x2 + 1
eExample 47 limx!1 p =?
x2 + 2x4 x
Solution
✓ ◆
2 1
x 1+ 2
x2 + 1 x
lim p = lim r !
x!1 x2 + 2x4 x x!1
1
x2 1+ 2
x3
1
= p
1+ 2
p p
Example 48 limx!1 ( x2 + x x2 + 5) =?

Solution

p p p p
p p ( x2 + x
x2 + 5)( x2 + x + x2 + 5)
lim ( x2 +x x2 + 5) = lim p p
x!1 x!1 x2 + x + x2 + 5
x2 + x x2 5
= lim q q
x!1
x( 1 + x1 + 1 + x52 )
5
x(1 x)
= lim q q
x!1 1 5
x( 1+ x + 1+ x2 )
1 1
= p p =
1+ 1 2

Example 49 p
lim ( x2 4x x) =?
x!1

16
Solution
p
p p ( x2 4x + x)
lim ( x2 4x x) = lim ( x2 4x x) p
x!1 x!1 ( x2 4x + x)
x2 4x x2
= lim p
x!1 x2 4x + x
4x
= lim ⇣q ⌘
x!1
x 1 x4 + 1
4
=
2
= 2

Example 50 p p
x + 1 + 2x + 1
lim =?
x!1 x

Solution
r r !
p 1 1
p p x 1+ + 2+
x + 1 + 2x + 1 x x
lim = lim
x!1 x x!1
r rx
1 1
1+ + 2+
x x
= lim p
x!1 x
= 0
1
Example 51 limx! 1 =0
x
Theorem 52 Let Pm (x) = am xm + am 1 xm 1 + ... + a0 and Qn (x) = bn xn +
bn 1 xn 1 +...+b0 be polynomials of degree m and n, respectively, so that am 6= 0
and bn 6= 0. Then
1. ⇢
1 am > 0
lim Pm (x) =
x!1 1 am < 0

2. 8 a 9
> m m=n >
Pm (x) < bn =
lim = ±1 m > n
x!±1 Qn (x) >
: >
;
0 m<n

x2 + 1
Example 53 limx!1 =0
3x3 4x + 5

17
3.2 Infinite Limits
A function whose values grow arbitrarily large can sometimes be said to have
an infinite limit. Since infinity is not a number, infinite limits are not really
limits at all, but they provide a way of describing the behaviour of functions
that grow arbitrarily large positive or negative.
1
Example 54 Find limx!0 if it exist.
x2

Solution As x approaches 0 from either side, the values of f (x) are positive
and grow larger and larger, so the limit of f (x) as x approaches 0 does not exist.
It is nevertheless convenient to describe the behaviour of f near 0 by saying that
1
f (x) approaches 1 as x approaches 0. We write limx!0 f (x) = limx!0 2 = 1.
x
2x
Example 55 Evaluate lim .
x!3+ x 3

Solution
2x
lim+ = +1
x!3 x 3
Example 56 Evaluate lim+ ln x.
x!0

Solution
lim ln x = 1
x!0+

bxc
Example 57 limx!0 =?
x

Solution

bxc 1
lim = lim = +1
x!0 x x!0 x
Example 58 Evaluate the following limits.
1. limx!1 (2x4 x2 8x) (answ. 1)
1 5
2. limt! 1( t + 2t3 t2 + 8) (answ. 1)
3
2x4 x2 + 8x 2
3. limx!1 4
(answ. )
5x + 7 5
p p
3x2 + 6 3
4. limx!1 (answ. )
5 2x 2

18
·
5. limx! 1
p
3x2 + 6
5 2x
(answ.
p

2
3
·)

3
6. limx!4 (answ. limit doesn’t exist)
(4 x)3
4x 8x2
7. limx!1 e2 (answ. 0)
✓ ◆
1
8. limt! 1 ln (answ. 1)
t2 5t

9. limx!1 arctanx (answ. )
2
✓ ◆
1 ⇡
10. limx!0 arctan (answ. )
x 2

4 The Formal Definition of Limit


Definition 59 A formal definition of limit
We say that f (x) approaches the limit L as x approaches a, and we write
limx!a f (x) = L if the following condition is satisfied:
for every number ✏ > 0 there exists a number > 0, possibly depending
on ✏, such that if 0 <| x a |< , then x belongs to the domain of f and
| f (x) L |< ✏.

Example 60 Show that limx!1 (5x 3) = 2.


Solution Set a = 1, f (x) = 5x 3 and L = 2 in the definition of limit.
Let ✏ > 0 be given. We must find > 0 so that if 0 <| x 1 |< then
| f (x) L |< ✏.
We find by working backward from the ✏-inequality:

| (5x 3) 2 |=| 5x 5 |=| x 1 |< .
5
Thus, we can take = ✏/5 and the implication above will be true. This
proves that limx!1 (5x 3) = 2.

Definition 61 Right limits


We say that f (x) has right limit L at a and we write limx!a+ f (x) = L if
the following condition is satisfied:
for every number ✏ > 0 there exists a number > 0, possibly depending on ✏,
such that if a < x < a+ , then x belongs to the domain of f and | f (x) L |< ✏.

The definition for a left limit is formulated in a similar way.

Definition 62 Limit at infinity


We say that f (x) approaches the limit L as x approaches infinity and we
write limx!1 f (x) = L if the following condition is satisfied:
for every number ✏ > 0 there exists a number R, possibly depending on ✏,
such that if x > R, then x belongs to the domain of f and | f (x) L |< ✏.

19
The definition for a limit at negative infinity is formulated in a similar way.

Definition 63 Infinite limits


We say that f (x) approaches infinity as x approaches a and we write limx!a f (x) =
1 if for every positive number B we can find a positive number , then x belongs
to the domain of f and f (x) > B.

Try to formulate the corresponding definition for the concept limx!a f (x) =
1.

5 Continuity
Definition 64 Continuity
A function f (x) is said to be continuous at an interior point c of its domain
if > F !s cont at C
- .
=

lim f (x) = f (c).


x!c

A function is said to be continuous on the interval [a, b] if it is continuous at


each point in the interval.

Note that this definition is also implicitly assuming that both f (c) and
limx!c f (x) exist. If either of these do not exist the function will not be con-
tinuous (discontinuous) at x = c .

Definition 65 Right and left continuity


We say that f is right continuous at c if limx!c+ f (x) = f (c).
We say that f is left continuous at c if limx!c f (x) = f (c).
p
Example 66 Let f (x) = x. The domain of f is [0, 1). We get
p
lim+ f (x) = lim+ x = 0 = f (0).
x!0 x!0

Therefore, f is right-continuous at 0.
In the above example, f is also continuous at every a > 0. Thus, it is
”continuous” at every a belonging to its domain, where ”continuous at 0” means
right-continuous at 0.

Theorem 67 Function f is continuous at c if and only if it is both right con-


tinuous and left continuous at c.

Definition 68 We say that f is continuous at a left endpoint c of its domain


if it is right continuous there.
We say that f is continuous at a right endpoint c of its domain if it is left
continuous there.

20
Example 69 Given the graph of f (x) , shown below, determine if f (x) is con-
tinuous at x = 2, x = 0 and x = 3.

Figure 3

Solution To answer the question for each point well need to get both the
limit at that point and the function value at that point. If they are equal the
function is continuous at that point and if they arent equal the function isnt
continuous at that point.
First x = 2. f ( 2) = 2 and limx! 2 f (x) doesn’t exist. The function value
and the limit arent the same and so the function is not continuous at this point.
This kind of discontinuity in a graph is called a jump discontinuity. Jump
discontinuities occur where the graph has a break in it as this graph does and
the values of the function to either side of the break are finite (i.e. the function
doesnt go to infinity).
Now x = 0 . f (0) = 1 and limx!0 f (x) = 1. The function is continuous at
this point since the function and limit have the same value.
Finally x = 3. f (3) = 1 and limx!3 f (x) = 0. The function is not contin-
uous at this point. This kind of discontinuity is called a removable disconti-
nuity. Removable discontinuities are those where there is a hole in the graph
as there is in this case.

From this example we can get a quick working definition of continuity. A


function is continuous on an interval if we can draw the graph from start to
finish without ever once picking up our pencil. The graph in the last example
has only two discontinuities since there are only two places where we would have
to pick up our pencil in sketching it.
In other words, a function is continuous if its graph has no holes or breaks
in it.

21
neceer haram
old
Bak bu araber

Date de
kanne


0 if x<0
Example 70 f (x) = Determine whether f is continuous at
x if x 0
x = 0 or not.

Solution Since limx!0+ f (x) = limx!0 f (x) = 0 = f (0) then f is contin-


uos at x = 0.
( (x(0 X > 0)
sin x ,
if x 6= 0
Example 71 f (x) = x Determine whether f is continuous
F(0) = 0 ,
0 if x = 0
at x = 0 or not.

e!m S!x
Solution f (0) = 0 and limx!0 f (x) = 1. Since limx!0 f (x) 6= f (0) then f
X)0 is not continuous at x = 0.

Theorem 72 If the functions f and g are both defined on an interval containing


and both are continuous at c, then the following functions are also continuous
at c:
1. the sum f + g and the di↵erence f g,
2. the product f.g,
3. the constant multiple kf , where k is any number,
4. the quotient f /g (provided g(c) 6= 0),
5. the nth root (f (x))1/n , provided f (c) > 0 if n is even.

Theorem 73 If f (x) is continuous at x = b and limx!c g(x) = b then,

lim f (g(x)) = f ( lim g(x)).


x!c x!c

In particular, if g is continuous at c (so b = g(c)), then the composition f g


is continuous at c:
lim f (g(x)) = f (g(c)).
r!x= ex3g(x) en s
x!c
=

Example 74 Evaluate the following limit.


l!m S!n x = 0
l!ms!nx
lim esinx - ex + 0 = e" = 1 X+0
x!0
eX !s a cont-Func
Solution Since we know that exponentials are continuous everywhere we can
use the theorem above. everywhere
espec!ally at X = 0
.
lim esinx = elimx!0 sinx = e0 = 1.
x!0
⇢ p
Example 75 f (x) =
ous at x = 1 or not.
x 1
x 1
if
if
x 1
x<1
Determine whether f is continu-
-ex
22
Solution

p
f (1) = 1 1=0
p
lim+ f (x) = lim+ x 1=0
x!1 x!1
lim f (x) = lim (x 1) = 0
x!1 x!1

f is continious at x = 1.

Example 76 Let f be the function given by



sin(n⇡x), 0 < x < 1
f (x) =
ln x, 1x<2
Discuss whether f is continuous on (0, 2).

Solution

f (1) = ln 1 = 0
lim f (x) = lim sin(n⇡x) = 0
x!1 x!1
lim f (x) = lim lnx = 0
x!1+ x!1+

Since
lim f (x) = 0 = f (1)
x!1
then f is continuous on (0, 2).
Example 77 For what value of b is
⇢ 3 1
x , x< 2
f (x) = 1
bx2 , x 2

continuous at every x?

Solution

✓ ◆
1 b
f =
2 4
b
lim+ f (x) = lim+ bx2 =
1
x! 2 x! 21 4
1
lim f (x) = lim x3 =
x! 12 x! 12 8
b 1 1
= )b= .
4 8 2

23
Example 78 For what values of a and b is
8
< 3 sin x, x<0

f (x) = a cos x + b, 0  x  2
:
sin x, x > ⇡2

continuous at every x?

Solution

lim f (x) = lim (acosx + b) = a + b


x!0+ x!0+
lim f (x) = lim 3sinx = 0
x!0 x!0
a+b=0

lim f (x) = lim ( sinx) = 1


x! ⇡
2
+ ⇡+
x! 2

lim f (x) = lim (acosx + b) = b


x! ⇡
2

x! 2

b= 1
a=1

Example 79 For what values of a and b is


8
< 2 cos x, x<0
f (x) = a cos x + b, 0  x  ⇡
:
sin x, x>⇡

continuous at every x?

Solution

lim f (x) = lim 2 cos x = 2


x!0 x!0
lim f (x) = lim a cos x + b = a + b
x!0+ x!0+

lim f (x) = lim a cos x + b = a+b


x!⇡ x!⇡
lim f (x) = lim sin x = 0
x!⇡ + x!⇡ +

24
a+b = 2
a+b = 0
a = 1
b = 1

Example 80 For what value of m is


8
< sgn( x + 1 ), x< 1
f (x) = |x+1|
: x2 + mx 1, x 1

continuous at every x?

Solution
x+1
lim sgn( ) = 1
x! 1 |x+1|
lim (x2 + mx 1) = 1 m 1= m
x! 1+
m = 1

Example 81 For what values of a and b is


8
< (x 1)3 x0
f (x) = axp+ b 0 < x < 1
:
x x 1

continuous at every x?

Solution

lim (x 1)3 = 1
x!0
lim ax + b = b
x!0+
b = 1

p
lim ax + b = lim+ x=1
x!1 x!1
a = 2

Example 82 Determine the continuity of given functions.

25
(a) ⇢ 1
x sin 2x , x 6= 0
f (x) =
0, x=0
Solution

f (0) = 0

1
lim f (x) = lim x sin =0
x!0 x!0 2x
f is continuous at x = 0.

(b)
f (x) = x2 + sgn(x2 1)

Solution

lim f (x) = lim x2 + 1 = 2


x!1+ x!1+
lim f (x) = lim x2 1=0
x!1 x!1

It follows that limx!1 f (x) does not exist and so f is not continuous at 1.

lim f (x) = lim x2 1=0


x! 1+ x! 1+

lim f (x) = lim x2 + 1 = 2


x! 1 x! 1

It follows that limx! 1 f (x) does not exist and so f is not continuous at
1.
Thus, f is not continuous at x = ±1 and f has jump discontinuities at
these points.
(c)
⇢ x2 1
x 1 , x 6= 1
f (x) =
2, x=1
Solution

f (1) = 2

26
x2 1
lim f (x) = lim
x!1 x!1 x 1
(x 1)(x + 1)
= lim
x!1 x 1
= lim (x + 1)
x!1
= 2

f is continuous at x = 1.

(d) 8
< x2 , x<1
f (x) = 2 x, x>1
:
0, x=1

Solution

lim f (x) = lim 2 x=1


x!1+ x!1+
lim f (x) = lim x2 = 1
x!1 x!1

limx!1 f (x) = 1 6= 0 = f (0). f has a removable discontinuity.

Theorem 83 Intermediate Value Theorem


Suppose that f (x) is continuous on [a, b] and let M be any number between
f (a) and f (b) . Then there exists a number c such that,
1. a < c < b,
2. f (c) = M.

All the Intermediate Value Theorem is really saying is that a continuous function
will take on all values between f (a) and f (b) . Below is a graph of a continuous
function that illustrates the Intermediate Value Theorem.

27
Figure 4

As we can see from this image if we pick any value, M , that is between the
value of f (a) and the value of f (b) and draw a line straight out from this point
the line will hit the graph in at least one point. In other words, somewhere
between a and b the function will take on the value of M . Also, as the figure
shows the function may take on the value at more than one place. Its also
important to note that the Intermediate Value Theorem only says that the
function will take on the value of M somewhere between a and b. It doesnt say
just what that value will be. It only says that it exists. These are important
ideas to remember about the Intermediate Value Theorem.
A nice use of the Intermediate Value Theorem is to prove the existence of
roots of equations as the following example shows.

Example 84 Show that p(x) = 2x3 5x2 10x + 5 has a root somewhere in
the interval [ 1, 2].

Solution
What were really asking here is whether or not the function will take on the
value p(x) = 0 somewhere between 1 and 2. In other words, we want to show
that there is a number c such that 1 < c < 2 and p(c) = 0 . However if we
define M = 0 and acknowledge that a = 1 and b = 2 we can see that these two
condition on c are exactly the conclusions of the Intermediate Value Theorem.
So, this problem is set up to use the Intermediate Value Theorem and in fact,
all we need to do is to show that the function is continuous and that M = 0 is
between p( 1) and p(2) (i.e. p( 1) < 0 < p(2) or p(2) < 0 < p( 1) and well
be done.
To do this all we need to do is compute, p( 1) = 8, p(2) = 19. So, we
have, 19 = p(2) < 0 < p( 1) = 8. Therefore M = 0 is between p( 1) and p(2)

28
and since p(x) is a polynomial its continuous everywhere and so in particular
its continuous on the interval [ 1, 2]. So by the Intermediate Value Theorem
there must be a number 1 < c < 2 so that, p(c) = 0. Therefore, the polynomial
does have a root between 1 and 2. For the sake of completeness here is a graph
showing the root that we just proved existed. Note that we used a computer
program to actually find the root and that the Intermediate Value Theorem did
not tell us what this value was.

Figure 5
✓ ◆
x2
Example 85 If possible, determine if f (x) = 20sin(x + 3)cos takes the
2
following values in the interval [0, 5].
a) s!ne and cos!ne funct!ons are cont .
everywhere
>
-
(a) Does f (x) = 10 ? So f(x) !s cont . everywhere , espec!ally on [0 , 5)

F(0) = 2 8224
.
,
f(5) = 19 7436
.

b) f(x) = -
10 ? E (b) Does f (x) = 10 ? F(0) < 10 < F(S)
Then CS t
by I UT
. .
there ex!stey . .

[0 5] but
,
F!s cont on
Solution 0 <5 and f(s) 10
.

, ,
=

- 10 !s not between Flo) and

#(5) Then ,
.
we can not apply So as with the previous example, were being asked to determine, if possible,
I V
. .
T

. if the functio takes on either of the two values above in the interval [0, 5]. First,
sowecannot sayanyth!natat lets notice that this is a continuous function and so we know that we can use
the Intermediate Value Theorem to do this problem.
Now, for each part we will let M be the given value for that part and then
well need to show that M lives between f (0) and f (5). If it does, then we can use
the Intermediate Value Theorem to prove that the function will take the given
value. So, since well need the two function evaluations for each part lets give
them here, f (0) = 2.8224 and f (5) = 19.7436.
Now, lets take a look at each part.

29
(a) In this case we’ll define M = 10 and we can see that, f (0) = 2.8224 <
10 < 19.7436 = f (5). So, by the Intermediate Value Theorem there must be a
number 0  c  5 such that f (c) = 10.
(b) In this part we’ll define M = 10. We now have a problem. In this
part M does not live between f (0) and f (5). So, what does this mean for us?
Does this mean that f (x) 6= 10 in [0, 5]? Unfortunately for us, this doesn’t
mean anything. It is possible that f (x) 6= 10 in [0, 5], but is it also possible that
f (x) = 10 in [0, 5]. The Intermediate Value Theorem will only tell us that c’s
will exist. The theorem will NOT tell us that c’s dont exist. In this case it is
not possible to determine if f (x) = 10 in [0, 5] using the Intermediate Value
Theorem.

Theorem 86 Extreme Value Theorem


Let f be a function that is defined and continuous on a closed and bounded
interval [a, b]. Then f attains its maximum and minimum in [a, b], that is, there
exist x1 , x2 such that f (x1 )  f (x)  f (x2 ) for all x 2 [a, b].

Remark 87 In the Extreme Value Theorem,


1. Closed intervals cannot be replaced by open intervals,
2. The assumption that f is continuous cannot be omitted.
⇢ 2
x , x 6= 0
Example 88 1. Let f (x) = For each real number a, deter-
1, x = 0
mine whether f is continuous or discontinuous at a.

| x |, 1x1
2. Let f : R ! R be the function given by f (x) =
1, otherwise
Discuss whether f is continuous on [ 1, 1].
8 2
< x ,
> x<1
3. Let f (x) = 1, 1  x<2
: 1,
>
x 2
x
Sketch the graph of f for x 2 [0, 5] and find all the point(s) in R at which
f is discontinuous.
x2 + x 2
4. Let f (x) = p .
1 x
(a) What is the domain of f ?
(b) Find limx!1 f (x).
(c) Can we define f (1) to make f continuous at 1? If yes, what is the
value?
5. Let p(x) = x5 6x4 3x3 + 5x2 + 7. Show that the equation p(x) = 0 has
a solution between 1 and 2.

30
References
[1] Robert A. Adams, Christopher Essex. Calculus: A Complete Course, Pear-
son, Canada, 2018.
[2] S.K. Chung, Understanding Basic Calculus, lecture notes, Department of
Mathematics, University of Hong Kong.
[3] Paul Dawkins, Calculus I, Paul’s Online Notes. Retrieved from
https://tutorial.math.lamar.edu/

31
Derivatives of Functions

Tangent Lines and Their Slopes


Let C be the curve that has equation y = f (x), where f is a function which is continuous on an interval J
containing x0 , and suppose that x0 is an interior point of J. If y0 = f (x0 ), then the point P (x0 , y0 ) is on
the curve C. If Q is a point on C di↵erent from P , then the line through P and Q is called a secant line
to the curve C.

If L is a line through P (x0 , y0 ) whose slope is the limit of the slopes of the secant lines through P and Q as
Q approaches P along C, then we say that L is tangent to C at P or L is the tangent line to the graph
of the function f (x) at P .

Since f (x) is a function, vertical lines can intersect C at most once. Since Q 6= P , Q must have a di↵erent
x coordinate from x0 . Therefore, Q = (x0 +h, f (x0 +h)), where h 6= 0. The slope of the line passing through
f (x0 + h) f (x0 )
P and Q is , and this expression is called Newton quotient or di↵erence quotient for
h
f at x0 . (h can be negative or positive)

Figure 1

Definition : Suppose that the function f is continuous at an interior point x0 of its domain and that
f (x0 + h) f (x0 )
lim = m exists. Then the straight line having slope m and passing through the point
h!0 h
P (x0 , f (x0 )) is called the tangent line to the graph of the function y = f (x) at P . An equation of this
tangent line is
y = m(x x0 ) + f (x0 ).

1
seğr" x2
Example. Find an equation of the tangent line to the curve y = x2 at the point (1, 1). y
=

Solution. The slope of the tangent line is


.....
(1 + h)2 1 h2 + 2h
m = lim = lim = lim (h + 2) = 0 + 2 = 2.
h!0 h h!0 h h!0 f(x) = X
P(xy yj)
, = (1 , 1)

= tangent
An equation of the tangent line is y 1 = 2(x 1), or equivalently, y = 2x 1. L ne

Remark : Suppose that f is a function that is continuous at an interior point x0 of its domain. Then,
using the substitution x = x0 + h, we obtain that

f (x0 + h) f (x0 ) f (x) f (x0 )


lim = lim .
h!0 h x!x0 x x0
Therefore, the slope m of the tangent line to the curve y = f (x) at the point P (x0 , f (x0 )) can also calculated
f (x) f (x0 )
as m = lim . Provided that the limit exists.
x!x0 x x0
2
Example. Find an equation of the tangent line to the hyperbola y = at the point ( 2, 1).
x
Solution. The slope of the tangent line is

l!m
2 x+2
f (x) f ( 2) +1
m= m = lim = lim x = lim x = lim 1 = 1
x! 2 x ( 2) x! 2 x+2 x! 2 x+2 x! 2 x 2
1 1
An equation of the tangent line is y ( 1) = (x ( 2)), or equivalently, y = x 2.
2 2
Definition : If the function f is continuous at x0 and if either

f (x0 + h) f (x0 )
lim = 1,
h!0 h
or
f (x0 + h) f (x0 )
lim = 1,
h!0 h
then the vertical line x = x0 is tangent to the graph of the function y = f (x) at (x0 , f (x0 )).

f (x0 + h) f (x0 )
Remark : If f is a function such that lim does not exist and is not 1 or 1, then the
h!0 h
graph of y = f (x) has no tangent line at x = x0 .

Example. Determine whether or not the graphs of each of the following functions has a tangent line at
x = 0. If the graph has a tangent line at x = 0, find its equation.

(a) f (x) =
p
3
x -
M
=
F
=!le
em =Ta = =

l!m En Tot =

to = 0

(b) f (x) = x2/3 x ot


+

so the tangant l ne !s the


,

vert!cal l!ne X =
0

2
i
i
Solution. (a) p
3
f (0 + h) f (0) h 0 1
lim = lim = lim 2/3 = 1
h!0 h h!0 h h!0 h
1
because if h is close to 0, then h2/3 is a small positive number so that 2/3 is a large positive number.
h
p
Thus, the graph of y = 3
x has a tangent line at x = 0 which is vertical, and its equation is x = 0.

(b)
f (0 + h) f (0) h2/3 0 1
lim = lim = lim 1/3
h!0 h h!0 h h!0 h

1 1
lim = 1 and lim = 1.
h!0 h1/3 h!0+ h1/3
f (0 + h) f (0)
So, the limit lim does not exist.
h!0 h
Therefore, there is no tangent line to the graph of the curve y = x2/3 .

The Derivative
Definition : The derivative of a function f is another function f 0 defined by

f (x + h) f (x)
f 0 (x) = lim
h!0 h
at all points x for which the limit exists (as a finite real number).

f (x0 + h) f (x0 )
The derivative of a function f at a number x0 is f 0 (x0 ) = lim if this limit exists.
h!0 h
f (x0 + h) f (x0 )
If f 0 (x0 ) exists, then we say that f is di↵erentiable at x0 . If the limit lim does not
h!0 h
exist, then we say that f is not di↵erentiable at x0 . The process of calculating the derivative f 0 of a given
function f is called di↵erentiation.

Remark : The value of the derivative of a function f at a point x0 can also be expressed as

f (x) f (x0 )
f 0 (x0 ) = lim
x!x0 x x0
provided that the limit exists, because under the substitution x = x0 + h we have

f (x0 + h) f (x0 ) f (x) f (x0 )


lim = lim .
h!0 h x!x0 x x0
f (x) f (x0 )
If lim does not exist, then f is not di↵erentiable at x0 .
x!x0 x x0

3
Example. Show that the derivative of the function f (x) = x3 is f 0 (x) = 3x2 .

Solution.
f (x + h) f (x) (x + h)3 x3
f 0 (x) = lim = lim
h!0 h h!0 h
3 2 2
x + 3x h + 3xh + h 3 x 3
= lim = lim (3x2 + 3xh + h2 )
h!0 h h!0
2 2 2
= 3x + 3x · 0 + 0 = 3x .

Thus, f 0 (x) = 3x2 .

Example. Show that the derivative of the function f (x) = sin x is f 0 (x) = cos x.

Solution.
f (x + h) f (x) sin(x + h) sin x sin x cos h + cos x sin h sin x
f 0 (x) = lim = lim = lim

h!0 h h!0 h

h!0 h
cos h 1 sin h
= lim sin x · + cos x ·
h!0 h h

cos h 1 (cos h 1)(cos h + 1) cos2 h 1 sin2 h


lim = lim = lim = lim
h!0 h h!0 h(cos h + 1) h!0 h(cos h + 1) h!0 h(cos h + 1)
✓ ◆
sin h 1 1
= lim · sin h · = 1·0· =0
h!0 h cos h + 1 2

cos h 1
So, lim = 0.
h!0 h
Therefore,

sin(x + h) sin x cos h 1 sin h


lim = sin x lim + cos x lim = sin x · 0 + cos x · 1 = cos x
h!0 h h!0 h h!0 h

Consequently, f 0 (x) = cos x.

Example. Prove that the function f (x) = |x| is not di↵erentiable at 0.

Solution.
f (x) f (0) |x| 0 |x|
lim = lim = lim
x!0 x 0 x!0 x x!0 x

|x| x
lim = lim = lim 1 = 1
x!0+ x x!0 x
+ x!0+

|x| x
lim = lim = lim ( 1) = 1
x!0 x x!0 x x!0

|x| |x| |x|


Since lim 6= lim , the limit lim does not exist. Thus, f (x) = |x| is not di↵erentiable at 0.
x!0+ x x!0 x x!0 x

Remark : The function f (x) = |x| is continuous at 0, but it is not di↵erentiable at 0.

4
The derivative of the function f (x) = |x| is


0 1, if x < 0
f (x) =
1, if x > 0.
Exercise. Show that the derivative of the function f (x) = ex is f 0 (x) = ex .

Example. Let
8 ✓ ◆
< 1
x2 sin , if x 6= 0
f (x) = x
:
0, if x = 0.
(a) Determine whether or not the function f is continuous at 0.

(b) Determine whether or not the function f is di↵erentiable at 0.

Solution.
✓ ◆ ✓ ◆
1 1
(a) 0  x2 sin 2
= x sin  x2 for all x 2 R \ {0} .
x x
lim 0 = 0 and lim x2 = 02 = 0.
x!0 x!0
✓ ◆
2 1
By the Squeeze Theorem, lim x sin = 0.
x!0 x
✓ ◆
2 1
So, lim x sin = 0. Since f (0) = 0, we have lim f (x) = f (0). Thus, f (x) is continuous at 0.
x!0 x x!0
✓ ◆
2 1
h sin ✓ ◆
f (0 + h) f (0) f (h) f (0) f (h) h 1
(b) lim = lim = lim = lim = lim h sin
h!0 h h!0 h h!0 h h!0 h h!0 h
✓ ◆
1
0  h sin  |h|
h
lim 0 = 0 and lim |h| = 0.
h!0 h!0
✓ ◆ ✓ ◆
1 1
By the Squeeze Theorem, lim h sin = 0. So, lim h sin = 0.
h!0 h h!0 h
Therefore, f (x) is di↵erentiable at 0 and f 0 (0) = 0.

Theorem : If a function f is di↵erentiable at x0 , then f is continuous at x0 .

Leibniz Notation
If y = f (x), we can use the dependent variable y to represent the function f (x), and we can denote the
derivative of f (x) with respect to x in any of the following ways:

dy df d
f 0 (x) = y 0 = = = f (x) = Dx y = Dx f (x) = Df (x)
dx dx dx
dy df d
The notations , , f (x) are called Leibniz notations for derivative.
dx dx dx

5
Di↵erentiation Formulas
1) Derivative of a Constant Function
If f (x) = c, where c is a real number, then f 0 (x) = 0.

2) The Power Rule


d n
If n 2 Z+ , then (x ) = nxn 1.
dx

3) The Constant Multiple Rule


d d
If c is a real constant and f is a di↵erentiable function, then cf (x) = c f (x).
dx dx

4) The Sum Rule


d d d
If f and g are di↵erentiable functions, then f + g is di↵erentiable and f (x) + g(x) = f (x) + g(x).
⇣ ⌘ dx dx dx
or (f + g)0 (x) = f 0 (x) + g 0 (x)

Remark : The sum rule can be extended to the sum of any number of functions. For instance, if f, g, h, k
are di↵erentiable functions, then
d d d d d
f (x) + g(x) + h(x) + k(x) = f (x) + g(x) + h(x) + k(x),
dx dx dx dx dx
or (f + g + h + k)0 (x) = f 0 (x) + g 0 (x) + h0 (x) + k 0 (x).

5) The Di↵erence Rule


d d d
If f and g are di↵erentiable functions, then f g is di↵erentiable and f (x) g(x) = f (x) g(x).
⇣ ⌘ dx dx dx
or (f g)0 (x) = f 0 (x) g 0 (x)

6) The Product Rule


d d d
If f and g are di↵erentiable functions, then f g is di↵erentiable and f (x)g(x) = f (x) g(x)+g(x) f (x).
⇣ dx ⌘ dx dx
or (f g)0 (x) = f (x)g 0 (x) + g(x)f 0 (x) or (f g)0 (x) = f 0 (x)g(x) + f (x)g 0 (x)

Remark : The product rule can be extended to the product of any number of functions. For instance,
if f1 , f2 , ..., fn are di↵erentiable functions, then
0
f1 f2 · · · fn (x) = f10 (x)f2 (x)f3 (x) · · · fn (x) + f1 (x)f20 (x)f3 (x) · · · fn (x) + · · · + f1 (x)f2 (x)f3 (x) · · · fn0 (x).

In particular, if f , g and h are di↵erentiable functions, then


0
f gh (x) = f 0 (x)g(x)h(x) + f (x)g 0 (x)h(x) + f (x)g(x)h0 (x).

6
7) The Quotient Rule
f
If f and g are di↵erentiable functions, then is di↵erentiable and
g
d d
d ⇣ f (x) ⌘ g(x) dx f (x) f (x) dx g(x)
= 2
dx g(x) g(x)
or ⇣ f ⌘0 f 0 (x)g(x) f (x)g 0 (x)
(x) = 2 ,
g g(x)
or ⇣ f ⌘0 g(x)f 0 (x) f (x)g 0 (x)
(x) = 2 ,
g g(x)
where g(x) 6= 0.

8) The Reciprocal Rule


d
1 d⇣ 1 ⌘ dx
f (x)
If f is a di↵erentiable function, then
f
is di↵erentiable and
dx f (x)
= 2 , whenever f (x) 6= 0.
f (x)
(This is a corollary of the Quotient Rule)

d n
Remark : Since (x ) = nxn 1 for every n 2 Z+ , using the Reciprocal Rule we obtain that
dx
d n
d n d⇣ 1 ⌘ dx
(x ) nxn 1
n 1
(x )= = 2 = = nx
dx dx xn xn x2n

for every n 2 Z+ .

9) The Power Rule (General Version)


d k
If k is any real number, then (x ) = kxk 1.
dx
Example. Find the derivative of each of the following functions.

(a) f (x) = x4/5


4 4 1 4 1 4
Solution. f 0 (x) = x 5 = x 5 =
5 5 5x1/5
p
(b) f (x) = x
p 1 1 1 1 1
Solution. f (x) = x = x1/2 , f 0 (x) = x 2 = x 1/2 = p
2 2 2 x
1
(c) f (x) =
x

7
1 1
Solution. f (x) = =x 1, so f 0 (x) = x 2 =
x x2
5
(d) f (x) = 2x4 + 6 + 8
x
Solution. f 0 (x) = 8x3 30x 7 + 0 = 8x3 30x 7

x
(e) f (x) =
x2 +1
1 · (x2 + 1) x · 2x 1 x2
Solution. f 0 (x) = =
(x2 + 1)2 (x2 + 1)2
p ⇣ 1 ⌘
Example. Let f (x) = x 2x + . Find f 0 (4).
x
✓ ◆ ✓ ◆
0 1 1 p 1
Solution. f (x) = p 2x + + x 2
2 x x x2
✓ ◆ ✓ ◆ ✓ ◆ ✓ ◆
0 1 1 p 1 1 1 1 1 1 95
f (4) = p 2·4+ + 4 2 = 8+ +2 2 =2+ +4 =
2 4 4 42 4 4 16 16 8 16

The Chain Rule


If a function g is di↵erentiable at x and a function f is di↵erentiable at g(x), then the composite function
F = f g defined by F (x) = f g(x) is di↵erentiable at x and F 0 (x) = f 0 g(x) g 0 (x).

dy dy du
In Leibniz notation, if y = f (u) and u = g(x) are both di↵erentiable functions, then = · .
dx du dx
d k k 1 0
Corollary : If k is any real number and g is a di↵erentiable function, then g(x) = k g(x) g (x).
dx
Example. Find the derivative of each of the following functions.

(a) f (x) = (x2 + 4x + 1)50

Solution. f 0 (x) = 50(x2 + 4x + 1)49 (2x + 4)


p
(b) f (x) = x3 + 9
p
Solution. Let f (x) = (g h)(x) where g(x) = x, h(x) = x3 + 9. Then f 0 (x) = g 0 h(x) h0 (x). So,
1 3x2
f 0 (x) = p · 3x2 = p .
2 x3 + 9 2 x3 + 9
(c) f (x) = (2x + 1)5 (4x3 x + 1)4

Solution.

f 0 (x) = 5(2x + 1)4 2 (4x3 x + 1)4 + (2x + 1)5 4 (4x3 x + 1)3 (12x2 1)
4 3 4 5 3 3 2
= 10(2x + 1) (4x x + 1) + 4(2x + 1) (4x x + 1) (12x 1)

8
⇣ 2x5 1 ⌘4
(d) f (x) =
x2 + 1
Solution.
✓ ◆3 ✓ 5 ◆0 ✓ 5 ◆3
0 2x5 1 2x 1 2x 1 10x4 (x2 + 1) (2x5 1) 2x
f (x) = 4 = 4
x2 + 1 x2 + 1 x2 + 1 (x2 + 1)2
8x(3x5 + 5x3 + 1)(2x5 1)3
=
(x2 + 1)5

Remark : If f is a function that is di↵erentiable at x0 , then the slope of the tangent line to the graph of
y = f (x) at the point x0 , f (x0 ) is m = f 0 (x0 ).
p ⇣ 4⌘
4 x
Example. Find an equation of the tangent line to the curve y = p at the point 1, .
x + 2x 3
p
4 x
Solution. Let f (x) = p .
x + 2x
✓ ◆
1 p p 1
4 · p ( x + 2x) 4 x p +2 p p p
0 2 x 2 x 2+4 x 2 8 x 4 x
Then, f (x) = p = p = p .
( x + 2x)2 ( x + 2x)2 ( x + 2x)2
4 4
The slope of the tangent line is m = f 0 (1) = 2 = .
3 9
4 4
An equation of the tangent line is y = (x 1) or 4x + 9y = 16.
3 9

Normal Line
Let P be a point on a curve C in the xy plane. If the curve C has a tangent line L at the point P , then
the straight line N through P perpendicular to L is called the normal line to the curve C at P .

If the tangent line L is horizontal,then the normal line N is vertical. Moreover, if L is vertical, then N is
horizontal.

If the tangent line L is neither horizontal nor vertical, and if the slope of L is m, then the slope of the normal
1
line N is .
m
Example. Find and equation of the normal line to the curve y = x4 + 2x2 x at the point (1, 2).

dy
Solution. = 4x3 + 4x 1
dx
dy
= 4 · 13 + 4 · 1 1=7
dx x=1
1
The slope of the normal line is .
7
1 1 15
An equation of the normal line is y 2= (x 1) or y = x+ .
7 7 7

9
EX : F(x) = S!n2X

F'(x) = cos2X . (2x)

Derivatives of Trigonometric Functions = 2COS2X

d
1) sin x = cos x tanx
dx EX : F(x) = 2

d tan Stanx)'
2) cos x = sin x F'(x) = 2 In 2 .

dx tanx
d => 2 .
enz .
secx
3) tan x = sec2 x = 1 + tan2 x
dx
d
4) cot x = csc2 x = 1 cot2 x
dx
d
5) sec x = sec x tan x
dx
d
6) csc x = csc x cot x
dx

Derivatives of Exponential and Logarithmic Functions


d x
1) e = ex
dx
d x
2) a = ax · ln a for every real number a > 0.
dx
d 1
3) ln x =
dx x
d 1
4) loga x = for every real number a > 0, a 6= 1.
dx x ln a
Remark :

ln x, if x > 0
ln |x| =
ln( x), if x < 0

So, we have
8
> 1
>
> ,
if x > 0
d < x
ln |x| =
dx >
> 1
>
: ( 1), if x < 0
x
1
=
x
for every nonzero real number x.

d 1
Thus, ln |x| = for every x 2 R \ {0}.
dx x

10
Derivatives of Hyperbolic Functions
d
1) sinh x = cosh x
dx
d
2) cosh x = sinh x
dx
d
3) tanh x = sech2 x
dx
d
4) coth x = csch2 x
dx
d
5) sech x = sech x tanh x
dx
d
6) csch x = csch x coth x
dx

Higher Order Derivatives


If f is a di↵erentiable function, then its derivative f 0 is also a function, so f 0 may have a derivative of its
own, denoted by f 00 . f 00 = (f 0 )0 .

The function f 00 is called the second derivative of f because it is the derivative of the derivative of f .

If y = f (x), the notations of the second derivative of f are as follows:

d2 y d2 f d2
y 00 = f 00 (x) = = = f (x) = Dx2 y = Dx2 f (x)
dx2 dx2 dx2
✓ ◆
d d d2
Note that f (x) = 2 f (x).
dx dx dx
The third derivative of f is the derivative of the second derivative of f and it is denoted by f 000 .
f 000 = (f 00 )0 .

If y = f (x), then alternative notations for the third derivative of f are

d3 y d3 f d3
y 000 = f 000 (x) = = = f (x) = Dx3 y = Dx3 f (x)
dx3 dx3 dx3
Derivatives of order greater than 3 are defined similarly. In general, the nth derivative of f where n 2 N,
n 2, is denoted by the derivative of the (n 1)th derivative of f and it is denoted by f (n) .
⇣ ⌘
0
f (n) = f (n 1) , f (3) = f 000 , f (2) = f 00 , f (1) = f 0 .

If y = f (x), then alternative notations for the nth derivative of f are

dn y dn f dn
y (n) = f (n) (x) = = = f (x) = Dxn y = Dxn f (x)
dxn dxn dxn
✓ n 1

d d dn
Note that f (x) = f (x), n 2 N, n 2.
dx dxn 1 dxn

11
1 ( 1)n n!
Example. Let f (x) = . Show that f (n) (x) = for every n 2 N.
1+x (1 + x)n+1
d 1
Solution. f 0 (x) = (1 + x) 1 =
dx (1 + x)2
( 1)n n!
So, f (n) (x) = is correct for n = 1.
(1 + x)n+1
( 1)n n! k
(k) (x) = ( 1) k! , where k is
Assume that the equation f (n) (x) = is correct for n = k, i.e., f
(1 + x)n+1 (1 + x)k+1
some positive integer.
✓ ◆
d d ( 1)k k! ( 1)k+1 (k + 1)!
f (k+1) (x)
= (k)
f (x) = = ( 1)k k! ( k 1)(1 + x) k 2 = .
dx dx (1 + x)k+1 (1 + x)k+2
( 1)n n!
Thus, f (n) (x) = for every n 2 N by induction.
(1 + x)n+1
Example. Let f (x) = e2x sin(3x). Find f 000 (x) and f 000 (0).

Solution. f 0 (x) = 2e2x sin(3x) + 3e2x cos(3x)

f 00 (x) = 4e2x sin(3x) + 6e2x cos(3x) + 6e2x cos(3x) 9e2x sin(3x) = 5e2x sin(3x) + 12e2x cos(3x)

f 000 (x) = 10e2x sin(3x) 15e2x cos(3x) + 24e2x cos(3x) 36e2x sin(3x) = 46e2x sin(3x) + 9e2x cos(3x)

f 000 (0) = 9e0 cos 0 = 9.

Example. Find the derivative of each of the following functions.


'

F'(x) =
cos (cosx) pos!x) .

(a) f (x) = sin cos2 x - = cos (cos"x) (-S!n 2x)


.

=
= cos (cos2x) S!nx .

Solution. f 0 (x) = cos cos2 x 2 cos x( sin x) = sin(2x) cos cos2 x


p ehx)t
(b) f (x) = ln x + e4x >
-
= (enx +
e4x)t ! (enx + e
*
Y)
f(x) =
1((nx +

= ( e)
✓ ◆
E e4x (ux))
t(enx e) (f
1 1 +
+

Solution. f 0 (x) = p + 4e4x =


+ .
.

2 ln x + e4x x eux

(c) f (x) = 2tan(


p
x) f(x)
tank
? enz (tan( * ))' tank
se)
.
2
enz
= .

>
-
2
.

enz Sech) IV)


=

<tan!k
.

p
.

=>
. . x

0 tan(
p
x) p 1 p sec2 ( x)
Solution. f (x) = 2 · ln 2 · sec2 ( x) · p = ln 2 · 2tan( x) 1 · p
2 x x
ex
=
=sen
(d) f (x) = log5 sec2 (6x) >
- f(x)

1 12
Solution. f 0 (x) = · 2 sec(6x) · sec(6x) tan(6x) · 6 = tan(6x)
sec2 (6x) · ln 5 ln 5

(e) f (x) =
sin3 (4x2 )
2x x !ns
Secr(x). +an16x

1
tan
=

3 sin2 (4x2 ) cos(4x2 )8x · 2x sin3 (4x2 ) · 2 48x2 sin2 (4x2 ) cos(4x2 ) 2 sin3 (4x2 )
Solution. f 0 (x) = =
4x2 4x2

12
f(x) =

et(4(enx) ! (enx) + esx+1 (Sx +


1))

1)
** +

! (44nx) ( + be
= .

(ln x)4 + e5x+1


(f ) f (x) =
ln 2
✓ ◆ ✓ ◆
0 1 3 1 5x+1 1 4(ln x)3 5x+1
Solution. f (x) = 4(ln x) · + e ·5 = + 5e
ln 2 x ln 2 x
p
(g) f (x) = 3 sin(ex + cot(2x))
1
Solution. f 0 (x) = (sin(ex + cot(2x))) 2/3 cos(ex + cot(2x))(ex 2csc2 (2x))
3
✓ ◆
x 1
(h) f (x) = cosh(4 ) + sinh
x4
✓ ◆ ✓ ◆ ✓ ◆
0 x x 1 1 4 1
Solution. f (x) = sinh(4 )4 ln + cosh 4x 5 = ln 4 · sinh(4 x )4 x cosh
4 x4 x5 x4
d
Example. Verify that tan x = sec2 x.
dx
✓ ◆
d d sin x cos x · cos x sin x · ( sin x) 1
Solution. tan x = = 2
= = sec2 x
dx dx cos x cos x cos2 x
buraya
-
kadar
Implicit Di↵erentiation 1 V!ze
.

Suppose that we have an equation of the form F (x, y) = 0, where F (x, y) denotes an expression involving
the two variables x and y, where y is a function of x (y depends on x).

Sometimes we can solve F (x, y) = 0 for y and so find explicit formulas for one or more functions y = f (x)
defined by the equation. Usually, however, we are not able to solve the equation F (x, y) = 0 for y. We can
still regard the equation F (x, y) = 0 as defining y as one or more functions of x implicitly even if we can
dy
not solve for these functions explicitly. Moreover, the derivative can be found by using the implicit dif-
dx
ferentiation technique. The idea of implicit di↵erentiate (both sides) of the given equation with respect
⇣ dy ⌘
to x, regarding y as a function of x having derivative y 0 or .
dx
dy VX
y
=

if y 2 = x.
A
Example. Find ↑
dx
Solution.
7
p dy 1 [

1st method: If y 0, then y = x. So, = p for x > 0.


dx 2 x
p dy 1 If y > 0 >y = =

If y < 0, then y = x. So, = p for x > 0.


dx 2 x y
& 1 1
=
=
=
=
-

k
dy 1 dx 25x 2Y V

In either case we have = for y 6= 0. In e!ther


dx 2y If y(8 > -x

de Ey
= =

cases
, =

2nd method: (Using implicit di↵erentiation)


==
d 2
dx
(y ) =
d
dx
(x) 2x2x
=dlyY =
dy dy 1
2y = 1. Thus, = for y 6= 0.
dx dx 2y
=>
2y . =

= = x

13
Example. Find an equation of the tangent line to the circle x2 + y 2 = 25 at the point (3, 4).

Solution.

1st method:
p
Since the point (3, 4) belongs to the lower half part of the circle x2 + y 2 = 25, we take y = 25 x2 .

dy 1 x
= p ( 2x) = p
dx 2 25 x2 25 x2
M

The slope of the tangent line is 15


-xz
-y
=

3
dy 3 3
m= =p =
dx x=3 25 3 2 4

-
An equation of the tangent line is >
3 -so
y ( 4) = (x 3)
4
or
3 25
y= x
4 4
2nd method: (Using implicit di↵erentiation)

d 2 d
(x + y 2 ) = (25)
dx dx
dy
2x + 2y =0
dx
dy 2x x
= =
dx 2y y
The slope of the tangent line is

dy 3 3
m= = =
dx (x,y)=(3, 4) 4 4
An equation of the tangent line is
3
y ( 4) = (x 3)
4
or
3 25
y= x
4 4
dy 2
Example. Find if y sin x + e2y = x3 + cos y.
dx
Solution.

d ⇣ 2
⌘ d
y sin x + e2y = x3 + cos y
dx dx
dy 2 dy dy
· sin x + y cos x + e2y · 4y · = 3x2 sin y ·
dx dx dx

14
dy ⇣ 2

sin x + 4ye2y + sin y = 3x2 y cos x
dx
dy 3x2 y cos x
Thus, =
dx sin x + 4ye2y2 + sin y
Example. Find y 00 if xy + y 2 = 2x.

Solution.

d d
xy + y 2 = (2x)
dx dx
dy dy
1·y+x + 2y =2
dx dx
dy
y+ (x + 2y) = 2
dx
dy 2 y
=
dx x + 2y
✓ ◆ ✓ ◆
d2 y d dy d2 y
= =
dx2 dx dx dx
x + 2y
✓ ◆
dy dy
dx (x + 2y) (2 y) 1 + 2
dx
=
(x + 2y)2
✓ ◆
2 y 2 y
(x + 2y) (2 y) 1 + 2 ·
x + 2y x + 2y
=
(x + 2y)2
x + 2y + 4 2y
y 2 + (y 2) ·
x + 2y
=
(x + 2y)2
✓ ◆
x+4
(y 2) 1 +
x + 2y
=
(x + 2y)2
(y 2)(x + 2y + x + 4)
=
(x + 2y)3
(y 2)(2x + 2y + 4)
=
(x + 2y)3
2xy + 2y 2 + 4y 4x 4y 8
=
(x + 2y)3
2(xy + y 2 2x) 8
=
(x + 2y)3
8
=
(x + 2y)3

15
Logarithmic Di↵erentiation
Suppose that y = f (x), where f is a di↵erentiable function. If f (x) contains products or quotients of many
h(x)
factors or if f (x) = g(x) for some functions g and h with g(x) > 0, then logarithmic di↵erentiation
0 dy
is useful for finding f (x) or equivalently . The steps in logarithmic di↵erentiation are as follows:
dx
1) If f (x) > 0 for all values of x, then take natural logarithms of both sides of the equation y = f (x) and
use the properties of logarithm to simplify.

If f (x) < 0 for some values of x, then take natural logarithms of both sides of the equation |y| = |f (x)| and
use the properties of logarithm to simplify.

2) Di↵erentiate implicitly with respect to x.

dy
3) Solve the resulting equation for .
dx
Example. Find the derivative of the function f (x) = xx for x > 0.

Solution.

1st method: Let y = xx . Then, ln y = ln(xx ) = x ln x.

d d
ln y = (x ln x)
dx dx
1 dy 1
· = ln x + x · = ln x + 1
y dx x
dy
= y(ln x + 1) = xx (ln x + 1)
dx
2nd method:
✓ ◆
d x d x ln x x ln x 1
(x ) = (e )=e 1 · ln x + x · = xx (ln x + 1)
dx dx x
(x + 1)(x + 2)(x + 3)
Example. Let f (x) = . Find f 0 ( 2).
x+4
Solution.

|x + 1| · |x + 2| · |x + 3|
|f (x)| =
|x + 4|
ln |f (x)| = ln |x + 1| + ln |x + 2| + ln |x + 3| ln |x + 4|

Di↵erentiating both sides of the last equation with respect to x we obtain

f 0 (x) 1 1 1 1
= + +
f (x) x+1 x+2 x+3 x+4
✓ ◆
0 1 1 1 1
f (x) = f (x) + +
x+1 x+2 x+3 x+4
(x + 2)(x + 3) (x + 1)(x + 3) (x + 1)(x + 2) (x + 1)(x + 2)(x + 3)
f 0 (x) = + +
x+4 x+4 x+4 (x + 4)2

16
( 1) · 1 1
f 0 ( 2) = 0 + +0 0=
2 2
cos x
Example. Find f 0 (x), if f (x) = ln x for x > 1.

Solution.

ln f (x) = cos x · ln(ln x)

f 0 (x) 1 1
= sin x · ln(ln x) + cos x · ·
f (x) ln x x
⇣ cos x ⌘
f 0 (x) = (ln x)cos x sin x · ln(ln x) +
x ln x
3/4
p
x x2 + 1
Example. Di↵erentiate y = .
(3x + 2)5
Solution.
3 1
ln y = ln x + ln(x2 + 1) 5 ln(3x + 2)
4 2
y 0 31 1 2x 3
= + · 2 5·
y 4x 2 x +1 3x + 2
✓ ◆
3 x 15
y0 = y + 2
4x x + 1 3x + 2
p ✓ ◆
0 x3/4 x2 + 1 3 x 15
y = +
(3x + 2)5 4x x2 + 1 3x + 2

Derivatives of Inverse Functions


Theorem : If f is a one-to-one di↵erentiable function with inverse function f 1 and f 0 f 1 (a) 6= 0, then
the inverse function f 1 is di↵erentiable at a and

1 0 1
f (a) = .
f 0 (f 1 (a))

Example. Let f (x) = 2x3 + 3x2 + 7x + 4. Given that f is one-to-one, find f 1 0 (4).

Solution. f (0) = 4. Therefore, f 1 (4) = 0. f 0 (x) = 6x2 + 6x + 7.

1 0 1 1 1 1
f (4) = = = = .
f 0 (f 1 (4)) f 0 (0) 6· 02 +6·0+7 7
Example. Let f (x) = 4x3 + 3 sin x + 2 cos x. Given that f is one-to-one, find f 1 0 (2).

Solution. f (0) = 2. Therefore, f 1 (2) = 0. f 0 (x) = 12x2 + 3 cos x 2 sin x.

1 0 1 1 1 1
f (2) = = = = .
f 0 (f 1 (2)) f 0 (0) 12 · 02 + 3 cos 0 2 sin 0 3

17
Example. Let f (x) = 2x + cos x. Given that f is invertible, find f 1 0 (⇡).

⇣⇡ ⌘ ⇡ ⇣⇡ ⌘ ⇡
Solution. f = 2 · + cos = ⇡. Therefore, f 1 (⇡) = .
2 2 2 2
f 0 (x) = 2 sin x.

1 0 1 1 1 1
f (⇡) = = ⇣⇡ ⌘ = ⇣⇡ ⌘ = = 1.
f 0 (f 1 (⇡)) f0 2 sin 2 1
2 2

Derivatives of Inverse Trigonometric Functions


d 1
1) arcsin x = p
dx 1 x2
d 1
2) arccos x = p
dx 1 x2
d 1
3) arctan x =
dx 1 + x2
d 1
4) arccot x =
dx 1 + x2
d 1
5) arcsec x = p
dx |x| x2 1
d 1
6) arccsc x = p
dx |x| x2 1
Example. Find the domain and the derivative of each of the following functions:
p
(a) f (x) = x arctan x.

(b) f (x) = arcsin(2x + 1) + x2 arccos(e2x ).

Solution.
p
(a) f (x) = x arctan x. The domain of f is [0, 1).
p
p 1 1 p x
f 0 (x) = arctan x+x· p 2 · p = arctan x +
1 + ( x) 2 x 2 (1 + x)
(b) f (x) = arcsin(2x + 1) + x2 arccos(e2x ).

1  2x + 1  1 () 2  2x  0 () 1  x  0.

1  e2x  1 () 0 < e2x  1 () 2x  ln 1 = 0 () x  0.

[ 1, 0] \ ( 1, 0] = [ 1, 0]

The domain of f is [ 1, 0].

18
1 1
f 0 (x) = q · 2 + 2x arccos(e2x ) + x2 · q · 2e2x
1 (2x + 1) 2
1 (e2x )2
2 2x2 e2x
= p + 2x arccos(e2x ) p
4x2 4x 1 e4x
1 2x2 e2x
= p + 2x arccos(e2x ) p
x2 x 1 e4x

Derivatives of Inverse Hyperbolic Functions


d 1
1) sinh 1 x = p
dx 1 + x2
d 1
2) cosh 1 x = p
dx x 2 1
d 1
3) tanh 1 x =
dx 1 x2
d 1
4) coth 1 x =
dx 1 x2
d 1
5) sech 1 x = p
dx x 1 x2
d 1
6) csch 1 x = p
dx |x| x2 + 1
Remark: The formulas for the derivatives of tanh 1 x and coth 1 x appear to be identical. But the domains
of these functions have no numbers in common: tanh 1 x is defined for |x| < 1, whereas coth 1 x is defined
for |x| > 1.

d 1 1
Example. Prove that sinh x= p .
dx 1 + x2
Solution.

1
1st method: Let y = sinh x. Then sinh y = x. If we di↵erentiate this equation implicitly with respect
to x, we get

dy
cosh y = 1.
dx
p
Since cosh2 y sinh2 y = 1 and cosh y 0, we have cosh y = 1 + sinh2 y, so

dy 1 1 1
= =p =p .
dx cosh y 2
1 + sinh y 1 + x2

19
⇣ p ⌘
1
2nd method: Since sinh x = ln x + x2 + 1 , we have

d d ⇣ p ⌘
1
sinh x = ln x + x2 + 1
dx dx
1 d ⇣ p ⌘
= p x + x2 + 1
x + x2 + 1 dx
✓ ◆
1 x
= p 1+ p
x + x2 + 1 x2 + 1
p
x2 + 1 + x
= ⇣ p ⌘p
x + x2 + 1 x2 + 1
1
= p .
x2 +1
d 1
Example. Find tanh (sin x).
dx
Solution. Using the Chain Rule, we have

d 1 1 d cos x 1
tanh (sin x) = sin x = = = sec x
dx 1 (sin x)2 dx cos2 x cos x

Derivatives of Parametric Functions


Sometimes the equation of a curve is not given in the form y = f (x) but given in parametric form as x = g(t),
y = h(t), where g and h are functions defined on an interval I and t 2 I is a real parameter. In the case that
the functions g and h are both di↵erentiable, we have
dy
dy h0 (t) ẏ
= dt = 0 =
dx dx g (t) ẋ
dt
dy
Example. Find if x = 5t and y = t2 6t + 1.
dx
dx dy dy dy/dt 2t 6
Solution. = 5 and = 2t 6. = = .
dt dt dx dx/dt 5
dy
Example. If x = t3 + 1 and y = t2 , calculate at t = 2.
dx
dx dy
Solution. = 3t2 and = 2t.
dt dt
dy dy/dt 2t 2
= = 2 = .
dx dx/dt 3t 3t
dy 2 2 1
Therefore = = = .
dx t=2 3t t=2 3 · 2 3
dy
Example. Find the value of at t = 1 if x = tet , y = t + et .
dx
dx dy
Solution. = et + tet = et (1 + t) and = 1 + et .
dt dt
dy dy/dt 1 + et
= = t .
dx dx/dt e (1 + t)

20
dy 1 + et 1+e
Therefore = t = .
dx t=1 e (1 + t) t=1 2e
✓ ◆ ✓ ◆
d2 y d dy d ẏ
Note. = = .
dx2 dx dx dx ẋ

Since is a function of t, we can write

✓ ◆ ⇢ ✓ ◆ ⇢ ✓ ◆
d dy d dy dt d ẏ 1 ẋÿ ẏẍ 1 ẋÿ ẏẍ
= = = · = .
dx dx dt dx dx dt ẋ dx (ẋ)2 ẋ (ẋ)3
dt
d2 y ẋÿ ẏẍ
Hence, 2
= .
dx (ẋ)3
d2 y
Example. Let x = a cos t and y = b sin t where a, b > 0 . Evaluate .
dx2
dx dy
Solution. = a sin t and = b cos t.
dt dt
dy dy/dt b cos t
= = .
dx dx/dt a sin t

✓ ◆ ✓ ◆ ✓ ◆
d2 y d dy d b cos t d b cos t dt
= = =
dx2 dx dx dx a sin t dt a sin t dx
b sin t · a sin t ( b cos t) · a cos t 1 ba
= 2
· =
(a sin t) a sin t (a sin t)3

d2 y
Example. Let x = 2t + 3 and y = 3t2 t. Calculate .
dx2
dx dy
Solution. = 2 and = 6t 1.
dt dt
dy dy/dt 6t 1
= = .
dx dx/dt 2
✓ ◆ ✓ ◆
d2 y d dy d 6t 1 dt 1 3
2
= = =3· =
dx dx dx dt 2 dx 2 2

Example. Let x = t sin t and y = t cos t. Find an equation of the tangent line to the curve at t = .
4
dy p ⇣ p ⌘
2 ⇡ 2
dy 1 · cos t + t · ( sin t) 2 + 4 2 4 ⇡
Solution. = dt = = p p = .
dx t= ⇡4 dx t= ⇡4 1 sin t + t · cos t t= ⇡4 2 ⇡
· 2 4+⇡
2 + 4 2
dt
4 ⇡
The slope of the tangent line is m = .
4+⇡
p p
⇡ ⇡ 2 ⇡ 2
At t = , we have x = and y = .
4 8 8 p ! p
4 ⇡ ⇡ 2 ⇡ 2
An equation of the tangent line is y = x + .
4+⇡ 8 8

21
Applications of Di↵erentiation

Theorem (Rolle’s Theorem) : Let a and b be real numbers such that b > a. Suppose that f is a
function that satisfies the following three hypotheses:

(i) f is continuous on the closed interval [a, b],

(ii) f is di↵erentiable on the open interval (a, b),

(iii) f (a) = f (b).

Then, there exists a number c in the open interval (a, b) such that f 0 (c) = 0.
⇣ ⇡⌘
Example. Show that the equation sin x = 2x 1 has a unique (exactly one) solution in the interval 0, .
2
Solution. Let f (x) = sin x 2x + 1.
h ⇡i
The function f is continuous on 0, because both sin x and 2x + 1 are continuous functions.
2
⇣ ⇡⌘
Moreover, f is di↵erentiable on 0, .
2
f (0) = sin 0 2 · 0 + 1 = 1 > 0.
⇣⇡ ⌘ ⇣⇡ ⌘ ⇡
f = sin 2· +1=2 ⇡ < 0.
2 2 2
⇣ ⇡⌘
Therefore, according to the Intermediate Value Theorem there exist a number c 2 0, such that f (c) = 0.
2
⇣ ⇡⌘
So the equation sin x = 2x 1 has a solution in the interval 0, .
2
⇣ ⇡⌘
Assume that the equation sin x = 2x 1 has two di↵erent solutions a and b in the interval 0, such that
2
b > a. Then f (a) = f (b) = 0.

Since f is continuous on [a, b] and di↵erentiable on (a, b), there exists a number in (a, b) such that f 0 ( ) = 0
by the Rolle’s Theorem.
⇣ ⇡⌘
f 0 (x) = cos x 2. Since cos x  1 we have f 0 (x)  1 2 = 1 for all x in 0, .
2
⇣ ⇡⌘
This is a contradiction. Therefore, the equation sin x = 2x 1 has a unique solution in the interval 0, .
2

22
Theorem (The Mean Value Theorem) : Let a and b be real numbers such that b > a. Suppose that
f is a function that satisfies the following hypotheses:

(i) f is continuous on the closed interval [a, b],

(ii) f is di↵erentiable on the open interval (a, b).

f (b) f (a)
Then, there exists a number c in the open interval (a, b) such that f 0 (c) = (or equivalently,
b a
f (b) f (a) = f 0 (c)(b a)).

Theorem : If f : (a, b) ! R is a di↵erentiable function such that f 0 (x) = 0 for all x in (a, b), then f is
constant on (a, b).

Corollary : If f : (a, b) ! R and g : (a, b) ! R are di↵erentiable functions such that f 0 (x) = g 0 (x) for
all x in (a, b), then f g is constant on (a, b), i.e., f (x) = g(x) + c for all x in (a, b), where c is a real constant.

Example. Show that sin x < x for all x > 0.

Solution. If x ⇡, then sin x  1 < ⇡  x.

Let x be a fixed real number such that 0 < x < ⇡.

Since the sine function is continuous and di↵erentiable everywhere on R, by the Mean Value Theorem, there
exists a number c 2 (0, x) such that

sin x sin 0 d
= sin x x=c
= cos c < 1.
x 0 dx

sin x
Thus, < 1.
x
So, sin x < x also for 0 < x < ⇡.

Consequently, sin x < x for all x > 0.


p x
Example. Show that 1+x<1+
for x > 0.
2
p x
Solution. Let us define the function f (x) = 1 + x 1. Suppose that x is a positive number. f
2
is continuous on [0, x] and di↵erentiable on (0, x). According to the Mean Value Theorem, there exists a
f (x) f (0)
number c 2 (0, x) such that = f 0 (c).
x 0
1 1 p 1 1
f 0 (c) = p < 0 because 1 + c > 1 so that p < .
2 1+c 2 2 1+c 2
p x
So f (x) < 0. Therefore, 1 + x < 1 + for all x > 0.
2

23
Theorem (The Generalized Mean Value Theorem) : If f and g are functions which are both
continuous on the interval [a, b] and both di↵erentiable on the interval (a, b), then there exists a number c
in (a, b) such that
f (b) f (a) f 0 (c)
= 0 .
g(b) g(a) g (c)

Example. Let the functions f (x) = x3 and g(x) = x2 + 4x be given. Find all possible values of the real
number c satisfying the Generalized Mean Value Theorem for the functions f and g on the closed interval
[ 1, 1].

Solution. The functions f (x) = x3 and g(x) = x2 + 4x are continuous on [ 1, 1] and di↵erentiable on
( 1, 1) since they are polynomials. According to the Generalized Mean Value Theorem,

f (1) f ( 1) f 0 (c)
= 0
g(1) g( 1) g (c)

for some real number c in ( 1, 1).

f (1) f ( 1) 1 ( 1) 1 f 0 (c) 3c2


= = , =
g(1) g( 1) 5 ( 3) 4 g 0 (c) 2c + 4
3c2 1
Thus, =
2c + 4 4
2 2 1
Therefore, 12c 2c 4 = 0 or (3c 2)(2c + 1) = 0 so that c = or c = , both of which belong to the
3 2
interval ( 1, 1).
2 1
All possible values of c are and .
3 2

24
Exercises ①3

S
1. Find the derivative of each of the following functions.

a. f (x) = (2x 3)4 (x2 +x+ 1)5 >


y = (2x -

34(x
u1n(2x-3) S(n(x2 + X + 1)
-

Iny =
.

✓ ◆3

=) /s!n(x
x2 + 1
b. f (x) = .

x2 1
p
c. f (x) = ln(ln( x)) - f(x) =

1 cosh x
d. f (x) =
1 + cosh x
e2x x3
e. f (x) = x2
e 1
e 2x x3
f. f (x) = x2
e 1
⇣ p ⌘
g. f (x) = arctan x x2 + 1
p 1x
h. f (x) = x2 1 · sec

i. f (x) = x tanh(sin2 (2x))

j. f (x) = (sin x)ln x

e x cos2 x
k. f (x) =
(x + 1)(x + 2)
2. Suppose that f is a di↵erentiable function defined on R such that f (1) = 2, f (2) = 3, f 0 (1) = 4, f 0 (2) = 5,
and f 0 (3) = 6. If g(x) = f (xf (xf (x))), find g 0 (1).

3. Let f (x) = xe x. Find f (1000) (x).

4. Show that the function y = e2x (2 cos(3x) sin(3x)) satisfies the equation y 00 4y 0 + y = 0.

x2 y2
5. Find an equation of the tangent line to the hyperbola = 1 at the point (x0 , y0 ).
a2 b2
6. Find an equation of the tangent line to the curve y = sin(4x) at the point (0, 0).

7. If x2 + xy + y 3 = 1, find the value of y 000 at the point where x = 1.

25
dy
8. Find if arctan(x2 y) = x + xy 2 .
dx a
9. Find all points on the curve x2 y 2 + xy = 2 where the slope of the tangent line is 1.

10. Find equations of the tangent line and normal line to the curve x2 + xy + 2y 3 = 4 at the point ( 2, 1).

11. Let f (x) = arctan x + ex . Given that f is one-to-one, find f 1 0 (1).

12. Let f : R ! R be a di↵erentiable function such 3  f 0 (x)  5. Show that 18  f (8) f (2)  30.

13. Prove that | sin a sin b|  |a b| for all a, b 2 R.

(2 . y) + (2x .
1 .

y')

+
x2+ 2xy + y3 =
u(1 1) ,

2x + (2y) + (2x .

y + 3y 2 y = 0

x3 x +
+
y
-
1 X= 1 2xy' + 3y2 y'
1 1
=
-
2x
11
-

2y

2x 2x + + 1 = m 2y' + 3 .

y' = -
2 -
2

Sy = -
U

y F(1) F'(1) (X 1) y' -


-

=
-
.
=

y -

2 =
6 (x 1)
-
y -
f(x) =
f(x) (X Xo) .
-

y 1
= (x 1)
-

y 6x
-

= -
4

(, 2)
*
f(x) =
e Fonks!y
↓ e
-
1 X= 0

F(x) =
e 2(2) (X Xo)
-

8+ 4 1 11
y -Yo
-
=
= M .
-

1
(2)
f(x) X) +
et 24
-
=

m
.
2

f(x) 2x2 +2 3 4 141


=
>
-
. +2 =

2
y -
f(x) = f'(x) (X X0) .
-

=MM .
Mn = -1

(e ) f(x) (X
E
-1
y y m
(0)
-
=
X
-
.

= -

-
2
11

y= z x +
t =
y =

=(2x +

1) y -
11 = - 1(X
14
-

2)

l!m

! 26
Bibliography

[1] Robert A. Adams, Christopher Essex. Calculus: A Complete Course, Pearson, Canada, 2018.

[2] S.K. Chung, Understanding Basic Calculus, lecture notes, Department of Mathematics, University of
Hong Kong.

[3] Paul Dawkins, Calculus I, Paul’s Online Notes. Retrieved from https://tutorial.math.lamar.edu/

[4] James Stewart, Calculus: Metric Version, 8th Edition, Cengage Learning, 2015.

l!m (x-E) . tanx =? 0 .


0

X+
E
e!n =

em
X+ +o

= to = 0
,

rex)

27
1 Extremum Values and Maximum Minimum
Problems
1.1 Increasing and Decreasing Functions

Recall Let f be a function in a domain D. We say that

 f is increasing if "x1 < x2 ) f (x1 ) < f (x2 ), for all x1 ; x2 2 D"


 f is decreasing if "x1 < x2 ) f (x1 ) > f (x2 ), for all x1 ; x2 2 D"

Theorem Let f be continuous on [a; b] and be di§erentiable in (a; b). Then,

 if f 0 (x) > 0 for all x 2 (a; b), then f (x) is increasing on (a; b)
 if f 0 (x) < 0 for all x 2 (a; b), then f (x) is decreasing on (a; b).

Example Let f (x) = x3  3x2 + 4. Find the intervals where f is increasing


and decreasing.
Solution f 0 (x) = 3x2 6x. We set f 0 (x) = 0. Then, 3x2 6x = 3x(x2) = 0.

x 0 2
0
f + 0 - 0 +
f % & %

So f is increasing in (1; 0) [ (2; 1) and f is decreasing in (0; 2).

1.2 Concavity and Ináection Points

Let f be twice di§erentiable on an interval I. Then,


00
 f is said to be concave up on I if f (x) > 0 for all x 2 I,
00
 f is said to be concave down on I if f (x) < 0 for all x 2 I.

Check the following Ögure:

1
Example Let f (x) = x3 ; g(x) = x4 and h(x) = x1=3 . Examine the concavity
of the each function.

00
If f (x) = x3 , then f 0 (x) = 3x2 and f (x) = 6x: The only point at which
00
f (x) = 0 or is undeÖned (f 0 is not di§erentiable) is at x = 0: If x < 0, then
00 00
f (x) < 0 so f is concave down. If x > 0, then f (x) > 0 so f is concave up.
00
If g(x) = x4 , then g 0 (x) = 4x3 and g (x) = 12x2 : The only point at which
00 00
g (x) = 0 or is undeÖned is at x = 0. If x < 0, then g (x) > 0 so g is concave
00
up. If x > 0, then g (x) > 0 so g is also concave up.
1 2=3 00 2 00
If h(x) = x1=3 , then h0 (x) =
x and h (x) =  x5=3 . h is not
00
3 900
deÖned if x = 0, but h > 0 for negative numbers and h < 0 for positive
numbers. So h changes concavity at (0; 0):
Remark The points where f changes its concavity character is called the
ináection points. In other words, second derivative of the function changes
sign at the ináection points.

Example In the following Ögure the concavity changes at points b and g.


At points a and h, the graph is concave up on both sides, so the concavity
does not change. At points c and f, the graph is concave down on both sides.
At point e, even though the graph looks strange there, the graph is concave
down on both sides-the concavity does not change.

1.3 Extremum Values


DeÖnition 1 Let f be a function and let x0 be in the domain of f: We say
that

2
 f has an absolute maximum value f (x0 ) at x0 if f (x0 )  f (x) for all x
in the domain of f:

 f has an absolute minimum value f (x0 ) at x0 if f (x0 )  f (x) for all x


in the domain of f:

 f has a local (relative) maximum value f (x0 ) at x0 if f (x0 )  f (x) for


all x in some open interval around x0 ;

 f has a local (relative) minimum value f (x0 ) at x0 if f (x0 )  f (x) for


all x in some open interval around x0 :

 Maximum and minimum values of a function are collectively referred


to as extreme values.

ext

4 :pdf
Fig 1: Local and absolute extremums

Remarks

 We will have an absolute maximum (or minimum) at x0 provided f (x0 )


is the largest (or smallest) value that the function will ever take on its
domain. On the other hand, f has a local maximum (or minimum)
value at x0 if f (x0 ) is the largest (or smallest) among the points su¢-
ciently close to x0 :

 An absolute maximum (or minimum) value can be attained at many


points. For example, f (x) = sin x has absolute maximum value 1 at

points x = + n for every integer n:
2
 A function need not have any absolute extreme values. For example
f (x) = 1=x becomes arbitrarily large as x approaches 0 from the right,
so has no absolute maximum. It doesnít have an absolute minimum
either. Even a bounded function may not have an absolute maximum
or minimum value. The function g(x) = x with domain speciÖed to
be the open interval (0; 1) has neither. Of course, if the domain of g

3
was extended to be the closed interval [0; 1] then g would have both a
maximum value, 1, and a minimum value, 0.

 A function can have only one absolute maximum (or minimum) value.
However, a function can have several local maximum (or minimum)
values. (see Fig 1)

Theorem 2 (Existence of extreme values) If the function f is continu-


ous on the bounded closed interval [a; b], then f must have an absolute max-
imum value and an absolute minimum value in [a; b].

Remark 3 If we have a continuous function on an interval [a; b] then we


are guaranteed to have both an absolute maximum and an absolute minimum
for the function somewhere in the interval. But the theorem doesnít tell us
where they will occur or if they will occur more than once. We have to pay
attention to the conditions of Theorem 2.
1
Example 4 f (x) = 2 doesnít have any absolute maximum on [1; 1] but it
x
has absolute minimum value 1. This is because
 f is not continuous at x = 0
1
(see Fig. 2) If we changed the interval to ; 1 ; then f has both absolute
2  
1
maximum and minimum (see Fig. 3) because f is continuous on ;1 .
2

5 4
y y
4
3
3
2
2
1
1

0
-1.0 -0.5 0.0 0.5 1.0 0.5 0.6 0.7 0.8 0.9 1.0
x x
1 1 1 
Fig. 2:y = on [1; 1] Fig. 3: y = on 2
;1
x2 x2

DeÖnition 5 Let f be a function and let x0 be a real number such that f is


deÖned on an open interval containing x0 : Recall that

4
0
 x0 is a singular point of f if f (x0 ) doesnít exist.
We say that
 x0 is a critical point of f if either
0 0
f (x0 ) = 0 or f (x0 ) doesnít exist (x0 is a singular point of f ):

The geometric interpretation of what is taking place at a critical point is


that the tangent line is either horizontal, vertical, or does not exist at that
point on the curve.
Example 6 Find all critical points of f (x) = x4  8x2 :
Solution: f is a polynomial and di§erentiable everywhere. Hence there
are no singular points.
f 0 (x) = 4x3  16x = 4x(x2  4):
f 0 (x) = 0 implies x = 0; x = 2; x = 2 are critical points.

y 120
100

80

60

40

20

-4 -3 -2 -1 1 2 3 4
x
Fig. 4: y = x4  8x2
x2 + 1
Example 7 Find all critical points of f (x) = :
x2  x  6
Solution: Using quotient rule,
x2 + 14x  1
f 0 (x) =  :
(x2  x  6)2
The derivative f 0 (x) doesnít exist at x = 2 and x = 3: However these
are NOT singular and hence critical points since f is not deÖned at these
points. Recall that in order for a point to be a critical point the function
must actually be deÖned at that point!
p
f 0 (x) = 0 ) x2 + 14x  1 = 0 ) x = 7  5 2
p
The critical points are x = 7  5 2:

5
8
y
6

-5 -4 -3 -2 -1 1 2 3 4 5
-2 x
-4

-6
x2 + 1
Fig. 5: y =
x2  x  6
p3
Example 8 Find all critical points of f (x) = x2 (2x  1) :

Solution: f (x) = 2x5=3  x2=3 and


10 2=3 2 1=3
f 0 (x) = x  x :
3 3
The derivative f 0 (x) doesnít exist at x = 0; thus x = 0 is a singular point
and a critical point.
10 2=3 2 1=3 2 1=3
f 0 (x) = x  x = x (5x  1) = 0 ) x = 1=5
3 3 3
The critical points are x = 1=5 and x = 0:

y 2

-1.4 -1.2 -1.0 -0.8 -0.6 -0.4 -0.2 0.2 0.4 0.6 0.8 1.0 1.2 1.4
x
-2

-4

p
3
Fig. 6: y = x2 (2x  1)

Remark 9 Figures 4,5 and 6 suggest that a function can take its local ex-
tremum values only at critical points or endpoints.

6
Theorem 10 (Locating Extreme Values) If the function f is deÖned on
an interval I and has a local maximum (or local minimum) value at point
x = x0 in I, then x0 must be either a critical point of f or an endpoint of I:

Remark 11 Theorem 10 tells us where to Önd extreme values. Although a


function cannot have extreme values anywhere other than at endpoints and
critical points, it need not have extreme values at such points. For example
the function f (x) = x3 has a critical point at x = 0; but this critical point is
neither a local minimum nor a local maximum. (see Fig.7)

y
20

10

-3 -2 -1 1 2 3
x
-10

-20

Fig. 7:y = x3

How to Find Absolute Extreme Values?


If a function f is deÖned on a bounded closed interval, Theorem 2 assures
us that f must have an absolute maximum value and an absolute minimum
value. Theorem 10 tells us how to Önd them. We need only to check the
values of f at any critical points and endpoints.

Example 12 Find the absolute maximum and minimum values of the func-
tion f (x) = x4  8x2 on the interval [4; 4] :

Solution: Since f is a polynomial, f is continuous on [4; 4] : From


Example 6, the critical points of f are x = 0; x = 2; x = 2: We need to
consider only the values of f at critical points x = 0; x = 2; x = 2 and
endpoints x = 4 and x = 4:

f (0) = 0; f (2) = f (2) = 16; f (4) = f (4) = 128:


The maximum value of f on [4; 4] is 128, at the endpoints x = 4
and x = 4 and the minimum value is 16, at the critical points x = 2
and x = 2:(see Fig. 4)
How to Find Local Extreme Values?

7
Theorem 10 tells us that a function can local extreme values only at
critical points or endpoints. But how can we decide whether a critical value
is a local maximum, local minimum or neither. The answer is in the next
theorem.

Theorem 13 (The First Derivative Test) Suppose that f is continuous


at a critical point x0 , and x0 is not an endpoint of the domain of f: Then,

i) If there exists an open interval (a; b) containing x0 such that f 0 (x) > 0 on
(a; x0 ) and f 0 (x) < 0 on (x0 ; b), then f has a local maximum value at
x0 . (see Fig 8a)

ii) If there exists an open interval (a; b) containing x0 such that f 0 (x) < 0
on (a; x0 ) and f 0 (x) > 0 on (x0 ; b), then f has a local minimum value
at x0 . (see Fig 8b)

iii) If f 0 (x) doesnít change sign at x0 ; then f has neither a maximum nor a
minimum value at x0 . (see Fig 8c)

Fig 8a: Local max

Fig 8b: Local min

8
Fig 8c: No max no min

Example 14 Find and classify local extreme values of f (x) = 27x  x3 :

Solution:
0
f (x) = 27  3x2 = 3(3  x)(3 + x) = 0 ) x = 3; x = 3:

f doesnít have singular points. The critical points are 3 and 3:

CP CP
x
-3 3
0
f - 0 + 0 -
f & min % max &
0
f is negative on the left of 3; and positive on the right of 3: By First
0
Derivative Test, x = 3 is a local minimum. f is positive on the left of 3; and
negative on the right of 3: By First Derivative Test, x = 3 is a local minimum.

150
y
100

50

-6 -4 -2 2 4 6
-50
x

-100

-150

Fig. 9: y = 27x  x3

Example 15 Find the local and absolute extreme values of f (x) = x4 4x3 +
5 on the interval [1; 4] :

9
Solution:
0
f (x) = 4x3  12x2 = 4x2 (x  3) = 0 ) x = 0; x = 3:

There is no singular values, so the critical points are 0 and 3:

EP CP CP EP
x
-1 0 3 4
0
f - 0 - 0 +
f abs. max & no min or max & abs. and loc. min %

From the table, x = 0 is not a local extremum and x = 3 is a local


minimum. For the absolute extremum values, we need to check the values at
the critical points and endpoints:

f (0) = 5; f (3) = 22; f (1) = 10; f (4) = 5:

The absolute maximum value of f is 10, at x = 1; and the absolute mini-
mum value is 22 at x = 3.

10
y

-1 1 2 3 4
x
-10

-20

Fig 10: f (x) = x4  4x3 + 5 on [1; 4]

Example 16 Find the local and absolute extreme values of f (x) = x + sin x
on the interval [0; 4] :

Solution:
0
f (x) = 1 + cos x = 0 ) x = (2n + 1) ; n 2 Z:

Thus the critical points are xn = (2n + 1) ; where n is an integer. Only 


and 3 are in the interval [0; 4] :
0
f (x) = 1 + cos x > 0 for x 2 [0; 4] ; x 6= ; x 6= 3:

10
EP CP CP EP
x
0  3 4
0
f + 0 + 0 +
f abs. min % no min or max % no min or max % abs. max
From the table, f doesnít have local extremum at the critical points 
and 3 . The absolute maximum is 4; at x = 4 and the absolute minimum
is 0; at x = 0: (see Fig. 11)

y 15

10

0
0 2 4 6 8 10 12 14
x
Fig. 11: y = x + sin x
p
3
Example 17 Find the local and absolute extreme values of f (x) = x2 (2x  1)
on the interval [1; 1] :

Solution: From Example 8,


10 2 2
f 0 (x) = x2=3  x1=3 = x1=3 (5x  1)
3 3 3
and the critical points are x = 1=5 and x = 0: The values at critical points
and endpoints are

f (1) = 3; f (0) = 0; f (1=5) = 0:205 20; f (1) = 1:

EP CP CP EP
x
1 0 1=5 1
0
f + 0 - 0 +
f abs. min % loc. max & loc. min % abs. max
From the table, x = 0 is a local maximum and x = 1=5 is a local minimum.
The absolute maximum value of f is 1, at x = 1; and the absolute minimum
value is 3 at x = 1. (see Fig. 12)

11
1
y

-1.0 -0.8 -0.6 -0.4 -0.2 0.2 0.4 0.6 0.8 1.0
x
-1

-2

-3
p
3
Fig. 12: y = x2 (2x  1) on [1; 1]
0
Theorem 18 (The Second Derivative Test) Suppose that f (x0 ) = 0:
00
i) If f (x0 ) < 0; then x0 is a local maximum of f:
00
ii) If f (x0 ) > 0; then x0 is a local minimum of f:
00
iii) If f (x0 ) = 0; no conclusion can be drawn; f may have a local maximum
at x0 or a local minimum, or it may have an ináection point instead.
Remark 19 For many functions the second derivative is more complicated
to calculate than the Örst derivative, so the First Derivative Test is likely to be
of more use in classifying critical points than is the Second Derivative Test.
Also note that the First Derivative Test can classify local extreme values that
occur at singular points as well as at critical points where the Örst derivative
is 0.
Example 20 Find and classify local extreme values of f (x) = xex :
Solution:
0 00
f (x) = ex + xex = (x + 1) ex ; f (x) = ex + (x + 1) ex = (x + 2) ex :
0
We begin by Önding critical points. f doesnít have singular points. f (x) = 0
implies x = 1 and the only critical point is x = 1: Using Second Derivative
00
Test, f (1) = e1 > 0 implies x = 1 is a local minimum of f: We can also
use First Derivative Test:

CP
x
-1
0
f - 0 +
f & min %
From the table, x = 1 is a local minimum of f: (see Fig 13)

12
x
-10 -9 -8 -7 -6 -5 -4 -3 -2 -1 0
0.0
y
-0.1

-0.2

-0.3

Fig 13: y = xex

4
Example 21 Find and classify local extreme values of f (x) = x + :
x
Solution: We have
0 4 00 8
f (x) = 1  2
; f (x) = 3
x x
0
The derivative f exists except x = 0; but x = 0 is not in the domain of f:
So, x = 0 is not a critical point.
0 4
f (x) = 1  = 0 ) x = 2
x2
00
The critical points are 2 and 2: Using Second Derivative Test, f (2) =
00
1 < 0 implies f has local maximum at x = 2 and f (2) = 1 > 0 implies
f has local minimum at x = 2: Similarly, we can use First Derivative Test
(do as an exercise).

10
y

-10 -8 -6 -4 -2 2 4 6 8 10
x
-5

-10
4
Fig 14: y = x +
x

13
Example 22 The functions

f (x) = x4 ; g(x) = x4 ; h(x) = x3

satisfy
0 00 0 00 0 00
f (0) = f (0) = 0; g (0) = g (0) = 0; h (0) = h (0) = 0:

However, f has a local minimum (Fig 15), g has a local maximum (Fig 16)
and h has neither a maximum nor a minimum (Fig 17) value at x = 0. This
00
example shows that if f (x0 ) = 0; no conclusion can be drawn in Second
Derivative Test.

y
1.0 x
-1.0 -0.5 0.0 0.5 1.0

y
0.5

-0.5

-1.0 -0.5 0.0 0.5 1.0


x -1.0

Fig 15: y = x4 Fig 16: y = x4

1.0
y
0.5

-1.0 -0.5 0.5 1.0


-0.5
x

-1.0

Fig 17: y = x3

Practise Problems
1) Find and classify all local extreme values of the given functions.

i) f (x) = x3  3x  2
x
ii) f (x) =
x2 + 1
p
iii) f (x) = x 2  x2

14
iv) f (x) = x  2 sin x
ln x
v) f (x) =
x
2) Find absolute extreme values of the given functions on given intervals.

i) f (x) = 8x3 + 81x2  42x  8 on [8; 2] : (Ans: Absolute Maximum : 1511


at x = 7; Absolute Minimum : -13:3125 at x = 1=4)
x+4
ii) f (x) = on [10; 0] : (Ans: Absolute Maximum : 0:5 at x =
2x2
+x+8 p
0; Absolute Minimum : 0:03128 at x = 4  3 2)
2
iii) f (x) = x2 (10  x) 3 on [2; 10:5] : (Ans: Absolute Maximum : 103:613 at
x = 7:5; Absolute Minimum : 0 at x = 10)

1.4 Maximum Minimum Problems

All maximum-minimum problems follow this same procedure:

i) Express the quantity Q to be maximized or minimized as a function of


one or more variables.

ii) If Q depends on n variables, where n > 1, Önd n  1 equations (con-


straints) linking these variables.

iii) Use the constraints to eliminate variables and hence express Q as a


function of only one variable. Determine the interval(s) in which this
variable must lie for the problem to make sense.

iv) Find the required extreme value of the function Q using the techniques
of previous section. Remember to consider any critical points and end-
points.

Example 23 Among all rectangles of given perimeter, show that the square
has the greatest area.

Solution: Suppose that the rectangle has width x and length y:

i) Maximize: Q = area = xy

ii) Constraint: perimeter P is given, so the constraint is 2x + 2y = P:

15
iii) Eliminating y we have y = 12 (P  2x) (You can also eliminate x): Now
Q becomes a function of one variable x :
1 1
Q(x) = xy = x (P  2x) = P x  x2 :
2 2
This is the function we will Önd the maximum. Our interval is 0  x 
1
2
P because the width canít be negative or greater than the half of the
perimeter.

iv)
0 1 P
Q (x) = P  2x = 0 ) x = :
2 4
P
The crtical point is x = 4 : At the endpoints x = 0 or x = P we have
Q = 0: So the maximum value comes from the critical point x = P4 :
If x = P4 then y = P4 and we have a square. The maximum area is
2
Q( P4 ) = P16 :

Example 24 We need to enclose a rectangular Öeld with a fence. We have


500 feet of fencing material and a building is on one side of the Öeld and
so wonít need any fencing. Determine the dimensions of the Öeld that will
enclose the largest area.

Solution:

i) Maximize: Q = xy

ii) Constraint: x + 2y = 500 (We are given 500 feet of fencing material)

iii) Eliminating x we have x = 500  2y (You can also eliminate y):Now Q


becomes a function of one variable y :

Q(y) = xy = y (500  2y) = 500y  2y 2 :

Interval: [0; 250] (If there is no length i.e. x = 0; then y = 250: Also the
width canít be negative i.e. y  0:)

16
iv)
0
Q (y) = 500  4y = 0 ) y = 125:
The crtical point is y = 125: At the endpoints y = 0 or y = 250 we have
Q = 0: So the maximum value comes from the critical point y = 125:
The maximum area is Q(125) = 31250 f t2 :
Example 25 We want to construct a box whose base length is 3 times the
base width. The material used to build the top and bottom cost $10=f t2 and
the material used to build the sides cost $6=f t2 . If the box must have a volume
of 50f t3 determine the dimensions that will minimize the -2
cost to build the
box.
Solution: 3w2h =
so

n
=
O h
bow+ 12wh + 36 wh

bowh
>
-
l zu
=

-
i) Minimize: Q = 10 (2lw) + 6 (2wh + 2lh) = 20lw + 12wh + 12lh:
s!n
- - -

ii) Constraints: l = 3w and 50 = lwh. (base length is 3 times the base width
and the box must have a volume of 50f t3 )
iii) We have l = 3w so 50 = lwh implies 50 = 3w2 h and eliminating (solving)
h gives h =
50
: Now we have only one variable w and Q becomes
bow
3w2
200 600 800 Gow+Pus .
so
Q(w) = 20lw + 12wh + 12lh = 60w2 + + = 60w2 + :
+ 3w)48 So
w w w 120
.

Interval: w > 0 but there is no upper limit to the width w that will give
&

0 3(w )
-

a box with volume of 50f t3 : So we donít have an interval of possible values =

but we can still Önd the minimum by using First Derivative Test or Second
Derivative Test. ? 3w 20 =
yu + 1-3w
4St
0 W
=
+ ( .

iv)
0 800 120w3  800
+w = 20
Q (w) = 120w  2 =
w w2
: w
= 20

The derivative doesnít exist at w = 0; but this is not a critical point


since Q(w) is not deÖned at this point. Also w = 0 doesnít give us a g
box.
r
5
3 20

-
0
Q (w) = 0 ) 120w3  800 = 0 ) w = = 1:8821
3

17 zu
(3w)
-gu-2
The only critical point is w = 1:8821 and we now have to verify that this
is in fact the value that will give the absolute minimum cost.
00 1600
Q (w) = 120 + > 0 for w > 0;
w3
so the graph of Q will be concave up around w = 1:8821 and hence Q takes
its minimum at w = 1:8821 (see Fig 18). The minimum cost is Q (1:8821) =
$637:60 and the dimensions for this minimal cost are
50
w = 1:8821; h = = 4:7050; l = 3w = 5:6463:
3w2

y
2000

1000

-5 -4 -3 -2 -1 1 2 3 4 5
x
-1000

800
Fig 18: Q(w) = 60w2 + w

Example 26 We want to construct a box with a square base and we only


have 10m2 of material to use in construction of the box. Assuming that all the
material is used in the construction process determine the maximum volume
that the box can have.

Solution:

i) Maximize: Q = lwh:

ii) Constraints: l = w and 10 = 2lw + 2lh + 2wh (the box has a square base
and we have 10m2 of material to use):

18
iii) We have l = w so 10 = 2lw + 2lh + 2wh implies 10 = 2w2 + 4wh
5  w2
and eliminating (solving) h gives h = : Now we have only one
2w
variable w and Q becomes
5  w2 1 
Q(w) = lwh = w = 5w  w3 :
2 2
iv) r
0 1 2
 5
Q (w) = 5  3w = 0 ) w =  = 1:2910:
2 3
We can ignore the negative critical point since w > 0:Now, as noted
in the previous example we got a single critical point, 1:2910, and so
this must be the value that gives the maximum volume (see the table
below).
EP CP
w
0 1:2910
0
f + 0 -
f % abs max &

The maximum volume is Q(1:2910) = 2:1517m3 :


Example 27 A manufacturer needs to make a cylindrical can that will hold
1500cm3 liquid. Determine the dimensions of the can that will minimize the
amount of material used in its construction.
Solution:

i) Minimize: Q = surf ace area = 2r2 + 2rh


ii) Constraint: volume = r2 h = 1500
1500
iii) Eliminating (solving) h from r2 h = 1500 gives h = : Now we have
r2
only one variable r and Q becomes
3000
Q(r) = 2r2 + 2rh = 2r2 + :
r

19
iv) r
0 3000 4r3  3000 3 750
Q (r) = 4r  2 = =0)r= = 6:2035:
r r2 
The only critical point is r = 6:2035: We have
00 6000
Q (r) = 4 + > 0 for r > 0;
r3
so the graph of Q will be concave up around r = 6:2035 and hence Q
takes its minimum at r = 6:2035: For the minimum material (or surface
area) the dimensions are
1500
r = 6:2035; h = = 12:4070:
r2

Example 28 We have a piece of cardboard that is 14 inches by 10 inches


and weíre going to cut out the corners as shown below and fold up the sides
to form a box, also shown below. Determine the height of the box that will
give a maximum volume.

Solution: Let the height of the box be h: So, the width/length of the
corners being cut out is also h and so the vertical side will have a ìnewî
height of 10  2h and the horizontal side will have a ìnewî width of 14  2h
(see Fig 19).

Fig 19: The box after cutting

20
i) Maximize: Q = V olume = h (10  2h) (14  2h) = 140h  48h2 + 4h3 :

ii) Constraint:No constraint, we already have Q as a function of one variable.

iii) No need to eliminate since we have only one variable. The interval is
0  h  5 because the height 10  2h  0 and the width 14  2h  0:

iv)
p
0 2 12  39
Q (x) = 14096h+12h = 0 ) h = ) h = 1:9183; h = 6:0817:
3
We can ignore the second critical value since the interval is 0  h  5:
So, the only critical value is 1:9183:Finally, since Q is deÖned and
continuous on 0  h  5 all we need to do is plug in the critical points
and endpoints into Q to determine which gives the largest volume.

Q(0) = 0; Q(1:9183) = 120:1644; Q(5) = 0:

So, if we take h = 1:9183 we get a maximum volume.

Example 29 A window is being built and the bottom is a rectangle and the
top is a semicircle. If there is 12 meters of framing materials what must the
dimensions of the window be to let in the most light?

Fig 20: The shape of the window

Solution: Letís rephrase the question as follows: we want a window in


the shape described above to have a maximum area (and hence let in the
most light) and have a perimeter of 12 m (because we have 12 m of framing
material).

i) Maximize: Q = area = 2hr + 12 r2

21
ii) Constraint: perimeter = 2h + 2r + r = 12
1
iii) Eliminating h we have h = 2
(12  2r  r) : Now we have only one
variable r and Q becomes
 
1 2 1 2 1
Q(r) = 2hr + r = (12  2r  r) r + r = 12r  2 +  r2 :
2 2 2

Interval: h  0 but no upper bound for h:

iv)
0 12
Q (r) = 12  (4 + ) r = 0 ) r = = 1:6803:
4+
00
The only critical value is 1:6803: We have Q (r) =  (4 + ) < 0 so
we can see that the maximum area must occur at this point. The
dimensions of the window be to let in the most light are

1
r = 1:6803; h = (12  2r  r) = 3:3606:
2
Example 30 Determine the area of the largest rectangle that can be inscribed
in a circle of radius 4.

Solution: Itís easiest to assume that the circle (and hence the rectangle)
is centered at the origin of a standard xy axis system. Doing this we know
that the equation of the circle will be x2 + y 2 = 16 and that the right upper
corner of the rectangle will have the coordinates (x; y) : This means that the
width of the rectangle will be 2x and the height of the rectangle will be 2y
as shown below

22
i) Maximize: Area of rectangle = Q = (2x) (2y) = 4xy

ii) Constraint: x2 + y 2 = 16 (the right upper corner of the rectangle will be


on the circle)
p
iii) Eliminating y from the constraint we have y =  16  x2 : Since the
point that weíre looking at is in the Örst quadrant we know that y
mustpbe positive and so we can take the ì+î part of this. Plugging
y = 16  x2 into Q we can write as a function of one variable:
p
Q(x) = 4xy = 4x 16  x2 :

Interval: 0  x  4 since we are assuming that x is in the Örst quadrant


and must stay inside the circle.

iv)
0 p 2x 64  8x2
Q (x) = 4 16  x2 + 4x p =p :
2 16  x2 16  x2
p
The singular points are x = 2 2 (makes the derivative zero) and x =
4 (singular points). We only want critical points that are in the interval
0p x  4 so that means that we have two critical points to deal with
2 2 and 4: Notice however that the second critical point is also one of the
endpoints of our interval. Now, Q is continuous on [0; 4] and all we need

23
to do is plug in the critical points and endpoints into Q to determine which
gives the largest area.
p
Q(0) = 0; Q(2 2) = 32; Q(4) = 0:
p
So, we can see that weíll getpthe maximum p area if x = 2 2 and and the
corresponding value of y is y = 16  x2 = 2 2: It looks like the maximum
area will be found if the inscribed rectangle is in fact a square.

Example 31 Find the shortest distance from the point (8; 1) to the curve
y = 1 + x3=2 :

Solution:

y 12
10

0 1 2 3 4 5
x
Fig 21: y = 1 + x3=2

Let P = (x; y) be the closest point on the curve y = 1 + x3=2 to the point
(8; 1) : The distance between P = (x; y) and (8; 1) is
q
D = (8  x)2 + (1  y)2

We want to minimize the distance D: Note that it is the same as minimizing


Q = (8  x)2 + (1  y)2 : The reason avoiding the square root is that taking
the derivative is simpler.

i) Minimize: Q = (8  x)2 + (1  y)2 = D2 :

ii) Constraint: y = 1 + x3=2 (P = (x; y) is a point on the curve y = 1 + x3=2 ).

iii) Eliminating y we have


 2
Q(x) = (8  x)2 +(1  y)2 = (8  x)2 + 1  1  x3=2 = x2 16x+64+x3 :

24
Interval: we donít have a bounded interval of x: Our interval is in fact the
domain of the function y = 1 + x3=2 ; [0; 1): (see Fig 21) So we are looking
for the absolute minimum of Q(x) on [0; 1):

iv)
0 8
Q (x) = 3x2 + 2x  16 = 0 ) x = 2; x =  :
3
8
The critical points are 2 and  3 : Only x = 2 is in our interval.

EP CP
x
0 2
0
f - 0 +
f & abs min %
From the table we see that we have the absolute minimum at x = 2:
The corresponding value of y is y = 1+23=2 = 3: 828 4:The closest point
is P = (2; 3: 828 4) : The shortest distance is
p p
D(2) = Q(2) = 44:

Example 32 Find the shortest distance from the origin to the curve x2 y 4 =
1:

Solution: Let P = (x; y) be the closest point on the curve x2 y 4 = 1 to


the origin. The distance between P = (x; y) and (0; 0) is
p
D = x2 + y 2 :

As we did in the previous example we can minimize Q = D2 = x2 + y 2 :

i) Minimize: Q = D2 = x2 + y 2 :

ii) Constraint: x2 y 4 = 1:

iii) Eliminating x2 from the constraint we have x2 = 1=y 4 and


1
Q(y) = x2 + y 2 = 4
+ y2:
y

Interval: we donít have a bounded interval of y: In fact y can be any real


number except 0:

25
iv)
4
0 p
6
Q (y) =
5
+ 2y = 0 ) y =  2:
y
p
The critical points are y =  6 2: (Note that y = 0 is not a critical point)

CPp CP
p
y 0
62 6
2
0
f - 0 + - 0 +
f & loc min % 1 & loc min %
p
From the table Q(y) has local minimum at y =  6 2: Note that we included
the point y = 0 not because it is a critical point but because the sign of the
0
derivative Q (y) changes around 0: Note also that

lim Q(y) = lim+ Q(y) = 1, lim Q(y) = 1;


y!0 y!0 y!1

and that our local minumums are also absolute maximums. (see Fig 22)

y 24
22
20
18
16
14
12
10
8
6
4

-5 -4 -3 -2 -1 0 1 2 3 4 5
x
1
Fig 22:y = x4
+ x2

The corresponding values of x are


1 2 1 1
x2 = 1=y 4 = p
6
= 2 3 ) x = 2 3 =  p
3
:
24 2
The closest points are
       
1 p 6 1 p 6 1 p
6 1 p
6
P1 =  p 3
; 2 ; P2 = p3
; 2 ; P3 =  p
3
;  2 ; P4 = p3
; 2
2 2 2 2
: The shortest distance is
p q p p p
6 6
D( 2) = Q( 2) = 22=3 + 21=3 = 1: 889 9:

26
Example 33 You are standing at the edge of a slow-moving river which is
one mile wide and wish to return to your campground on the opposite side
of the river. You can swim at 2 km=h and walk at 3 km=h. You must Örst
swim across the river to any point on the opposite bank. From there walk to
the campground, which is one mile from the point directly across the river
from where you start your swim. What route will take the least amount of
time ?

Solution:

Fig 23: The diagram for Example 33

Recall that if travel is at a CONSTANT rate of speed, then

(distance traveled) = (rate of travel)(time elapsed)


or

D = RT
so that time elapsed is
D
T = :
R
We want to minimize the total TIME elapsed:

27
T = (swim time) + (walk time)
= (swim distance)=(swim rate) + (walk distance)=(walk rate)
p
1 + x2 1  x
= + :
2 3
Taking derivative
p
0 x 1 3x  2 1 + x2
T (x) = p  = p
2 1 + x2 3 6 1 + x2
and the critical points are the roots of the equation
0 x 1 p
T (x) = p  = 0 ) 3x  2 1 + x2 = 0
2 1 + x2 3
 
) 9x2 = 4 1 + x2
4
) x2 =
5
2
) x =  p  0:89
5
x =  p25 are the critical points. Since 0  x  1 (see Fig 23) we ignore
the negative critical point.

EP CP EP
y p2
0 5
1
0
f - 0 +
f & loc min %

We only need to check the critical point x = p2 and endpoints:


5
p
5 2 2
T (0) = = 0:833 33; T ( p ) = 0:706 01; T (1) = = 0:707 11:
6 5 2

The minimum time elapsed is 0:706 01 hour.


Practise Problems
1) Two positive numbers have sum 7. What is the largest possible value
for their product? (Ans: 49=4)
2) Two nonnegative numbers have sum 60. What are the numbers if the
product of one of them and the square of the other is maximal? (Ans: 20
and 40)

28
3) The sum of two nonnegative numbers is 10. What is the smallest value
of the sum of the cube of one number and the square of the other? (Ans:
71:45)
4) Among all rectangles of given area, show that the square has the least
perimeter.
5) Among all isosceles triangles of given perimeter, show that the equi-
lateral triangle has the greatest area.
6) Find the area of the largest rectangle that can be inscribed in a semi-
circle of radius R if one side of the rectangle lies along the diameter of the
semicircle. (Ans: R2 )
7) A rectangle with sides parallel to the coordinate axes is inscribed in
the ellipse
x2 y 2
+ 2 = 1:
a2 b

Find the largest possible area for this rectangle. (Ans: 2ab)
8) A billboard is to be made with 100 m2 of printed area and with margins
of 2 m at the top and bottom and 4 m on each side. Find the outside
dimensions
p of the billboard p if its total area is to be a minimum. (Ans: width
8 + 10 2 m, height 4 + 5 2 m)
9) What is the length of the shortest line segment having one end on
the
 p x-axis,
 the other
p end on the y-axis, and passing through the point
9; 3 ? (Ans: 8 3)
10) Find the dimensions of the largest right-circular cylinder
q that can be
2R
inscribed in a sphere of radius R. (Ans: height p 3
; radius 23 R)
11) A box with square base and no top has a volume of 4 m3 . Find the
dimensions of the most economical box. (Ans: base 2 m  2 m; height 1 m)
12) We are going to fence in a rectangular Öeld. If we look at the Öeld
from above the cost of the vertical sides are $10=f t, the cost of the bottom
is $2=f t and the cost of the top is $7=f t. If we have $700 determine the
dimensions of the Öeld that will maximize the enclosed area. (Ans: lenght
350
9
, width 35
2
)
2
13)Find the
q point(s)
 on the curve
q x= 32y that are closest to (4; 0) :
(Ans:  154
; 27
8
and  15 4
;  27 8
)
14) A line through the point (2; 5) forms a right-triangle with the x-axis
and y-axis in the 1st quadrant. Determine the equation of the line that will
minimize the area of this triangle. (Ans: y =  52 x + 10 )

29
References
[1] Adams, R. A. and Essex, C., Calculus: A complete course, 9th Edition,
Pearson Canada, 2018.

[2] Dawkins, P., Paulís Online Notes, https://tutorial.math.lamar.edu/Classes/CalcI/DerivAppsI

[3] Chung, S. K., Understanding Basic Calculus, CreateSpace Independent


Publishing Platform, 2014.

30
1 CURVE SKETCHING

Recall Let f be a function in a domain D. We say that


f is increasing if ”x1 < x2 → f (x1 ) < f (x2 ), for all x1 , x2 ∈ D”
f is decreasing if ”x1 < x2 → f (x1 ) > f (x2 ), for all x1 , x2 ∈ D”
Theorem Let f be continuous on [a, b] and be differentiable in (a, b). Then,
if f → (x) > 0 for all x ∈ D, then f (x) is increasing on D
if f → (x) < 0 for all x ∈ D, then f (x) is decreasing on D.
Check Figure 1.

Figure 1

Concavity and Inflection points


Let f be twice differentiable on an interval I. Then,
!!
f is said to be concave up on I if f (x) > 0 for all x ∈ I,
!!
f is said to be concave down on I if f (x) < 0 for all x ∈ I.
Note that inflection points are the points where the function change concavity.

The following table gives the shape of the graph of f corresponding to the four cases determined by the signs of f → and
!! !!
f . For example, first row first column corresponds to that both f → and f are positive: the figure indicates that the
graph goes up and bends up. Check Figure 2.

1
Figure 2

ASYMPTOTES

In the previous sections, we have learned some applications of derivatives which are necessary for curve sketching. First
and second order derivative analysis will be applied in this section. Also, we need the behavior of the function around
infinity.

If the interval is unbounded, i.e. if the function is defined for −∞ ≤ x ≤ ∞ then you can’t compute the values f (a)
and f (b), but instead you should compute limx→a f (x) = ∓∞.

An asymptote of a curve y = f (x) that has an infinite branch is called a line such that the distance between the point
(x, f (x)) lying on the curve and the line approaches zero as the point moves along the branch to infinity.
Asymptotes can be vertical, oblique (slant) and horizontal. A horizontal asymptote is often considered as a special
case of an oblique asymptote.

1. Horizontal Asymptotes

If the distance between the graph of a function and some fixed line approaches zero as a point on the graph moves
increasingly far from the origin, we say that the graph approaches the line asymptotically and that the line is an
asymptote of the graph.

Definition A line y = b is a horizontal asymptote of the graph of a function y = f (x) if either

lim f (x) = b or lim f (x) = b.


x→∞ x→−∞

5x2 +8x−3
Example: The curve f (x) = 3x2 +2 is given. It has the line y = 5/3 as a horizontal asymptote on both the right
and the left because

lim f (x) = 5/3 and lim f (x) = 5/3


x→∞ x→−∞

Vertical Asymptotes

2
The straight line x = a is a vertical asymptote of the graph of the function y = f (x) if at least one of the following
conditions is true:
lim+ f (x) = ∓∞ lim− f (x) ∓ ∞
x→a x→a

In other words, at least one of the one-sided limits at the point x = a must be equal to infinity.
A vertical asymptote occurs in rational functions at the points when the denominator is zero and the numerator is not
equal to zero (i.e. at the points of discontinuity of the second kind). For example, the graph of the function y = x1 has
the vertical asymptote x = 0 . Check Figure 3.
lim f (x) = ∓∞ lim f (x) ∓ ∞
x→0+ x→0−

20.12.2020 https://www.math24.net/wp-content/uploads/2018/10/asymptote-of-hyperbolic-function.svg

https://www.math24.net/wp-content/uploads/2018/10/asymptote-of-hyperbolic-function.svg 1/1

Figure 3

Oblique(Slant) Asymptotes

The straight line y = kx + b is called an oblique (slant) asymptote of the graph of the function y = f (x) as x → ∞ if

lim (f (x) − (kx + b)) = 0


x→∞

A straight line y = kx + b is an asymptote of a function y = f (x) as x → ∞ if and only if the following two limits are
finite:

lim f (x) = k and lim (f (x) − kx) = b


x→∞ x x→∞

Check Figure 4.

3
20.12.2020 https://www.math24.net/wp-content/uploads/2018/10/definition-of-oblique-asymptote.svg

https://www.math24.net/wp-content/uploads/2018/10/definition-of-oblique-asymptote.svg 1/1

Figure 4

STEPS FOR SKECTHING GRAPHS

The following steps are taken in the process of curve sketching:

1. Domain
Find the domain of the function and determine the points of discontinuity (if any).
2. Intercepts
Determine the x− and y− intercepts of the function, if possible. To find the x−intercept, we set y = 0 and solve the
equation for x. Similarly, we set x = 0 to find the y−intercept.

4. Asymptotes

Find the vertical, horizontal and oblique (slant) asymptotes of the function.
5. Intervals of Increase and Decrease

Calculate the first derivative and find the critical points of the function. (Remember that critical points are the points
where the first derivative is zero or does not exist.) Determine the intervals where the function is increasing and
decreasing using the First Derivative Test.Use the First or Second Derivative Test to classify the critical points as local
maximum or local minimum.

7. Concavity/Convexity and Points of Inflection

!!
Using the Second Derivative Test, find the points of inflection (at which f = 0. Determine the intervals where the
function is concave up and concave down.

4
8. Graph of the Function.

EXAMPLES

1. Sketch f (x) = (x + 2)2 (x − 1).

The function is defined for all x ∈ R. Consequently, this function has no vertical asymptotes. Check for oblique (slant)
asymptotes. Calculate the slope of the asymptote:
(x+2)2 (x−1)
lim f (x) = lim = lim (x3 + 3x2 − 4) = lim (x2 + 3x − x4 ) = +∞.
x→∓∞ x x→∓∞ x x→∓∞ x→∓∞

This indicates that the function has also no oblique asymptotes. Then, it has no horizontal asymptotes.

Determine the points of intersection of the graph with the coordinate axes: f (0) = −4 and f (x) = 0 → (x+2)2 (x−1) =
0 → x = −2, x = 1.

We find the critical points by setting the first derivative equal to zero:
d
f → (x) = dx ((x + 2)2 (x − 1)) = 2(x + 2)(x − 1) + (x + 2)2 . Then, f → (x) = 0 → 3x(x + 2) = 0 → x = 0, x = −2.

The derivative changes its sign as shown in Figure 3(a). Therefore, x = −2 is the maximum point, and x = 0 is the
minimum point. The function has the following values at these extrema points: f (−2) = 0, f (0) = −4
Find the second order derivative:

!! !!
f (x) (x) = (3x(x + 2))→ → f (x) (x) = 6x + 6.
So the function is strictly concave up for x < −1 and strictly concave down for x > −1. Hence, x = −1 is the inflection
point, and f (−1) = −2. Check Figure 5.

20.12.2020 https://www.math24.net/wp-content/uploads/2018/10/signs-of-function-and-derivatives2.svg

https://www.math24.net/wp-content/uploads/2018/10/signs-of-function-and-derivatives2.svg 1/1

Figure 5

The graph is in Figure 6.

5
20.12.2020 https://www.math24.net/wp-content/uploads/2018/10/sketching-graph-of-function2.svg

https://www.math24.net/wp-content/uploads/2018/10/sketching-graph-of-function2.svg 1/1

Figure 6

1
2. Sketch f (x) = 1+x2

The function is defined for all real values of x ∈ R. Consequently, it has no vertical asymptotes. Since

1
lim f (x) = lim =0
x→∓∞ x→∓∞ 1 + x2

then the graph of the function has horizontal asymptote y = 0, that is the x-axis is the horizontal asymptote.

It is obvious that the function has no roots and positive for all x. At the point x = 0, its value is f (0) = 1.

1 −2x
The first derivative is f → (x) = ( 1+x →
2) = (1+x2 )2 . So there is only one critical point, f → (x) = 0 → x = 0.
Calculate the second order derivative :

!! −2(1+x2 )2 +2x.2(1+x2 ) 6x2 −2 !! √


f (x) = (1+x2 )4 = (1+x2 )3 . Then, f (x) = 0 → x = ∓ 3. Check Figure 7.

6
20.12.2020 https://www.math24.net/wp-content/uploads/2018/10/sgns- of -funct on- and- der vatves3. svg

https://www.math24.net/wp-content/uploads/2018/10/sgns- of -funct on- and- der vatves3. svg 1/1

Figure 7

The graph is in Figure 8.

20.12.2020 https://www.math24.net/wp-content/uploads/2018/10/sketchng- gr aph- of -funct on3. svg

https://www.math24.net/wp-content/uploads/2018/10/sketchng- gr aph- of -funct on3. svg 1/1

Figure 8

(x−1)3
3. Sketch f (x) = x2 .

The domain of the function is x ⇒= 0. The function has a discontinuity at x = 0. By calculating one-sided limits, we
obtain:

(x − 1)3
lim− f (x) = lim− = −∞
x→0 x→0 x2

7
(x − 1)3
lim f (x) = lim = −∞
x→0+ x→0− x2

It follows that x = 0 is a vertical asymptote. Check for slant asymptotes:

f (x) (x − 1)3
k = lim = lim =1
x→∞ x x→∞ x3
(x − 1)3 3 1
b = lim (f (x) − kx) = lim ( − x) = lim (−3 + − 2 ) = −3
x→∞ x→∞ x2 x→∞ x x

Consequently, there is an oblique asymptote given by the equation f (x) = x − 3.

The first order derivative is


(x − 1)3 → (x − 1)2 (x + 2)
f → (x) = ( ) =
x2 x3
So, the critical points are x = 0, x = 1, x = −2.
The function strictly decreases on the interval (−2, 0) and strictly increases on the intervals (−∞, −2), (0, 1) and (1, ∞).

The second order derivative is

!! (x − 1)2 (x + 2) → 6(x − 1)
f (x) = ( 3
) =
x x4
!! !! !!
It is seen that f = 0 at x = 1, and f < 0 to the left of this point and f > 0 to the right. Thus, the function is
convex upward on the intervals (−∞, 0), (0, 1) and is convex downward on the interval (1, ∞). Check the Figure 9.

The point x = 1 is an inflection point and y(1) = 0.

20.12.2020 https://www.math24.net/wp-content/uploads/2018/10/sgns- of -funct on- and- der vatves9. svg

https://www.math24.net/wp-content/uploads/2018/10/sgns- of -funct on- and- der vatves9. svg 1/1

Figure 9

The graph is in 10.

8
20.12.2020 https://www.math24.net/wp-content/uploads/2018/10/sketchng- gr aph- of -funct on9. svg

https://www.math24.net/wp-content/uploads/2018/10/sketchng- gr aph- of -funct on9. svg 1/1

Figure 10

4. Sketch f (x) = x ln x

The function is defined for x > 0. Determine the x−intercepts: f (x) = 0 → x = 1.

Look for the vertical asymptote at x = 0: Using L’Hopital’s rule, we can write the right-sided limit in the form

ln x 1/x
lim x ln x = lim = lim = lim − x = 0
x→0+ x→0+ 1/x x→0 −1/x2
+ x→0+

ln x 1/x
Also, lim x ln x = lim = lim+ −1/x 2 = −∞.
x→∞ x→∞ 1/x x→0

The function has neither vertical, nor horizontal asymptotes. Consider the possible oblique asymptote as x → ∞:

x ln x
lim = lim ln x = ∞
x→∞ x x→∞
.
So there are no oblique asymptotes. Take derivative:

1 1
f → (x) = ln x + x. = 0 → ln x = −1 → x =
x e
.
The critical point is x = 1e . The derivative changes sign from negative to positive at x = 1e . Therefore,
1
x= e is a point of local minimum.

!! !!
Next, f (x) = x1 . So, f (x) > 0. The function is concave up everywhere.
The graph is in Figure 11.

9
21.12.2020 https://www.math24.net/wp-content/uploads/2019/05/graph-curve-sketching3.svg

https://www.math24.net/wp-content/uploads/2019/05/graph-curve-sketching3.svg 1/1

Figure 11

x
5. Sketch f (x) = x2 −1

Find the domain of the function: x2 − 1 ⇒= 0 → x ⇒= ∓1.

f (x) = 0 → x = 0. Make sure that it has vertical asymptotes at x = ∓1:

x
lim − f (x) = lim − = −∞
x→−1 x→−1 x2 −1

x
lim f (x) = lim =∞
x→−1+ x→−1− x2 −1

x
lim f (x) = lim = −∞
x→1− x→−1− x2 −1

x
lim f (x) = lim =∞
x→1+ x→−1− x2 −1

Check for horizontal asymptotes: y = 0 is the horizontal asymptote.

x
lim f (x) = lim =0
x→∓∞ x→∓∞ x2 −1

Now, take derivative:

x x2 + 1
f → (x) = ( ) →
= −
x2 − 1 (x2 − 1)2

We see that f → (x) < 0 that is the function is decreasing everywhere in its domain.

The second order derivative is

10
!! x2 + 1 → 2x3 + 6x
f (x) = (− 2 2
) = 2
(x − 1) (x − 1)3
!!
So, f (x) = 0 → x = 0. Check the Figure 12.

21.12.2020 https://www.math24.net/wp-content/uploads/2019/05/sign-chart-curve-sketching16.svg

https://www.math24.net/wp-content/uploads/2019/05/sign-chart-curve-sketching16.svg 1/1

Figure 12

The graph is in Figure 13.

21.12.2020 https://www.math24.net/wp-content/uploads/2019/05/graph-curve-sketching17.svg

https://www.math24.net/wp-content/uploads/2019/05/graph-curve-sketching17.svg 1/1

Figure 13

!
6. Sketch f (x) = 3
x2 (x + 1)

11
The function is defined on the entire real axis. Its graph crosses the x−axis at x = 0 and x = −1. The function is
positive on the intervals (−1, 0) and (0, +∞) and negative on the interval (−∞, −1).

Since the function is continuous everywhere, it does not have a vertical asymptote. We calculate the coe”cients of the
oblique (slant) asymptote:

! ! !
3
x2 (x + 1) 3
x3 (1 + 1/x) 3
1 + 1/x
k = lim = lim = lim =1
x→∓∞ x x→∓∞ x x→∓∞ 1

! x3 + x2 − x3 1
b = lim (f (x) − kx) = lim 3
x2 (x + 1) − x = lim ! ! ! =
x→∓∞ x→∓∞ 3 x→∓∞ (x3 + x2 )2 + 3 (x3 + x2 )x3 + 3 (x6 3

Consequently, there is an oblique asymptote given by the equation y = x + 1/3. First order derivative is

3x + 2
f → (x) = !
3 3 x(x + 1)2

It follows from here that the function has the following critical points: x = 0, x = −1, x = −2/3.

Second order derivative is

!! −2
f (x) = !
9 x (x + 1)5
3 4

Check the Figure 14.

21.12.2020 https://www.math24.net/wp-content/uploads/2018/10/signs-of-function-and-derivatives7.svg

https://www.math24.net/wp-content/uploads/2018/10/signs-of-function-and-derivatives7.svg 1/1

Figure 14

The graph is in Figure 15.

12
21.12.2020 https://www.math24.net/wp-content/uploads/2018/10/sketchng- gr aph- of -funct on7. svg

https://www.math24.net/wp-content/uploads/2018/10/sketchng- gr aph- of -funct on7. svg 1/1

Figure 15

References
[1] Robert A. Adams, Christopher Essex. Calculus: A Complete Course, Pearson, Canada, 2018.
[2] S.K. Chung, Understanding Basic Calculus, lecture notes, Department of Mathematics, University of Hong Kong.
[3] Paul Dawkins, Calculus I, Paul’s Online Notes. Retrieved from https://tutorial.math.lamar.edu/
[4] James Stewart, Calculus: Metric Version, 8th Edition, Cengage Learning, 2015.
[5] https://www.math24.net/

13
1. Indeterminate Forms
sinx
Recall that limx!0 = 1. We can’t see this by substituting x = 0
x
into the function (sinx)/x because both sinx and x are zero at x = 0.
0
We call (sinx)/x an indeterminate form of type at x = 0. The
0
limit of such an indeterminate form can be any number. For example
sinx sin2 x sin2 x
lim = 1, lim = 0, lim = 1.
x!0 x3 x!0 x x!0 x2

We can collect indeterminate forms into 7 groups:


 h1i
0 ⇥ 0⇤ ⇥ 0⇤
, , [0.1] , [1 1] , 0 , 1 , [11 ] .
0 1
How to Evaluate Indeterminate Forms? 
0
1) We can evaluate many indeterminate forms of type by can-
0
celling common factors or using limit rules.
Example 1.

x2 + x 2 0
lim 2
x! 2 x + 5x + 6 0
(x + 2)(x 1)
= lim
x! 2 (x + 2)(x + 3)

x 1
= lim = 3.
x! 2 x + 3

Example 2.

1 cosx 0
lim
x!0 x2 0
1 (1 2sin2 (x/2))
= lim
x!0 x2
1 sin2 (x/2) 1
= lim =
x!0 2 (x/2)2 2
2) We will use L’Hospital’s
 Rules for evaluating limits of indetermi-
0 h1i
nate forms of the types and .
0 1
Theorem 3 (L’Hospital’s Rules). Suppose the functions f and g are
di↵erentiable on the interval (a, b) and g 0 (x) 6= 0 there. Suppose also
that
• limx!c f (x) = limx!c g(x) = 0 or ± 1,
f 0 (x)
• limx!c 0 = L (L is finite or ± 1),
g (x)
1
2

for a < c < b. Then,


f (x)
lim = L.
x!c g(x)
Similar results hold for the one-sided limits at endpoints and also for
a = 1 and b = 1.

Remark 4. • Note that in applying L’Hospital’s Rule we calcu-


late the quotient of the derivatives, not the derivative of the
quotient.
• It is possible that the limit of the quotient of derivatives may
still be indeterminate, in which case a second application of
L’Hospital’s Rule can be made. Such applications may be strung
out until a limit can finally be extracted.

sin(⇡x)
Example 5. Evaluate limx! 4 .
x2 16
Solution:

sin(⇡x) 0
lim 2
x! 4 x 16 0
(sin(⇡x))0
= lim
x! 4 (x2 16)0
⇡cos(⇡x) ⇡
= lim =
x! 4 2x 8
2sinx sin(2x)
Example 6. Evaluate limx!0 .
2ex 2 2x x2
Solution:

2sinx sin(2x) 0
lim x 2
x!0 2e 2 2x x 0
2cosx 2cos(2x)
= lim
x!0 2ex 2 2x 
cosx cos(2x) 0
= lim x
x!0 e 1 x 0

sinx + 2sin(2x) 0
= lim x
x!0 e 1 0
cosx + 4cos(2x)
= lim = 3.
x!0 ex
ln(3x)
Example 7. Evaluate limx!1 .
x2
3

Solution:
ln(3x) h 1 i
lim
x!1 x2 1
1/3x
= lim
x!1 2x
1
= lim =0
x!1 6x2

x2
Example 8. Evaluate limx! 1 .
e1 x
Solution:
x2 h1i
lim
x! 1 e1 x 1
2x h1i
= lim
x! 1 e1 x 1
2
= lim =0
x! 1 e1 x

x2 sin(1/x)
Example 9. Evaluate limx!0 .
sinx
Solution:

x2 sin(1/x) 0
lim
x!0 sinx 0
2xsin(1/x) cos(1/x)
= lim
x!0 cosx
The last limit doesn’t exist. In this case we can NOT apply L’Hospital’s
Rule but we can still use other limit techniques. Using Squeeze Theo-
rem
x2 sin(1/x)
lim = 0,
x!0 sinx
since
x2 x2 sin(1/x) x2
 
sinx sinx sinx
and
x2 ⇣ ⌘⇣ x ⌘
lim = lim x lim =0
x!0 sinx x!0 x!0 sinx

 3) Weh can convert the indeterminate forms [1 1] and [0.1] to


0 1 i
or .
0 1
✓ ◆
1 1
Example 10. Evaluate limx!0+ .
x sinx
4

Solution:

✓ ◆
1 1
lim [1 1]
x!0+ x sinx

sinx x 0
= lim+
x!0 xsinx 0

cosx 1 0
= lim+
x!0 sinx + xcosx 0
sinx 0
= lim+ = = 0.
x!0 2cosx xsinx 2 0

Example 11. Evaluate limx!0+ xlnx.

Solution:

lim xlnx [0.1]


x!0+
lnx h 1 i
= lim+
x!0 1/x 1
1/x
= lim+
x!0 1/x2
= lim+ ( x) = 0.
x!0

Example 12. Evaluate limx! 1 xex .

Solution:

lim xex [1.0]


x! 1

ex 0
= lim
x! 1 1/x 0
x

e 0
= lim
x! 1 1/x2 0

ex 0
= lim
x! 1 2/x3 0
x

e 0
= lim 4
x! 1 6/x 0
5

Clearly this choice of quotient is getting us nowhere! We have to rewrite


the quotient the other way around.
lim xex [1.0]
x! 1
x
= lim
x! 1 1/ex
x h1i
= lim
x! 1 e x 1
1
= lim = 0.
x! 1 e x

0 00 1
4) We can reduce the indeterminate forms [0 ] , [1 ] , [1 ] to or
h1i 0
by taking logarithms of the expressions involved.
1
Example 13. [10 ] Evaluate limx!1 x1/x .
1
Solution: Let y = x1/x . Taking logarithm we have lny = lnx.
x
lnx h1i
lim lny = lim
x!1 x!1 x 1
1/x
= lim = 0.
x!1 1

We know that y = elny .


lim x1/x = lim y = lim elny = elimx!1 lny = e0 = 1
x!1 x!1 x!1
2
Example 14. [11 ] Evaluate limx!0 [cos(2x)]1/x .
2
Solution: Let y = [cos(2x)]1/x . Taking logarithm we have
1
lny = lncos(2x).
x2

lncos(2x) 0
lim lny = lim
x!0 x!0 x2 0
2sin(2x)
cos(2x)
= lim
x!0 2x
tan(2x)
= lim 2 = 2
x!0 2x
2
lim [cos(2x)]1/x = lim y = lim elny = elimx!0 lny = e 2 .
x!0 x!0 x!0
p
Example 15. [00 ] Evaluate limx!0+ x x
.
6
p
x
Solution: Let y = x . Taking logarithm we have
p
lny = xlnx.
p
lim+ lny = lim+ xlnx [0.1]
x!0 x!0
lnx h 1 i
= lim+
x!0 1 1
p
x
lnx
= lim+ 1/2
x!0 x
1/x
= lim+ = lim 2x1/2 = 0.
x!0 1 3/2 x!0+
x
2
p
lim+ x x = lim+ y = lim+ elny = elimx!0+ lny = e0 = 1.
x!0 x!0 x!0
Practise Problems
Evaluate each of the following limits.
sin(2z) + 7z 2 2z
1) limz!0 (Ans: 7)
z 2 (z + 1)2
x2 + e4x
2) limx!1 (Ans: 1)
2x ex
3) limx!1 x ln 1 + x3 (Ans: 3)
4) limx!0+ x2 ln (4x2 ) (Ans: 0)
5) limx!0 x1 xe1ax (Ans: a)
2
6) limx!0+ (csc x)sin x (Ans: 1)
7) limx!0 (1 + tan x)1/x (Ans: 1)
References
[1] Adams, R. A. and Essex, C., Calculus: A complete course, 9th Edition, Pearson
Canada, 2018.
[2] Dawkins, P., Paul’s Online Notes, https://tutorial.math.lamar.edu/Classes/CalcI/DerivAppsIntro.aspx.
[3] Chung, S. K., Understanding Basic Calculus, CreateSpace Independent Pub-
lishing Platform, 2014.
1 INTEGRATION
1.1 Indefinite Integrals
Definition 1 Given a function, f (x) , an anti-derivative of f (x) is any function F (x) such that
0
F (x) = f (x).

Example 2 Let f be a function such that f (x) = x2 + 4 cos x. Then


F1 (x) = 2x  4 sin x + 2
F2 (x) = 2x  4 sin x  3
F3 (x) = 2x  4 sin x + .
F1 , F2 and F3 are anti-derivatives of f.

Definition 3 If F (x) is any anti-derivative of f (x) then the most general anti-derivative of f (x) is called
an indefinite integral and denoted,
Z
f (x)dx = F (x) + c, where c is any constant.
Z
In this definition the is called the integral symbol, f (x) is called the integrand, x is called the integration
variable and the c is called the constant of integration.
The process of finding the indefinite integral is called integration or integrating f (x) . If we need to be
specific about the integration variable we will say that we are integrating f (x) with respect to x.

1.2 Some Special Integral Formulas


Z
1
1. xn dx = xn+1 + c, for all n 6= 1
n+1
Z
1
2. dx = ln |x| + c
x
Z
3. ex dx = ex + c
Z
ax
4. ax dx = + c, a > 0
ln a
Z
5. sin x dx =  cos x + c
Z
6. cos x dx = sin x + c
Z
7. tan x dx =  ln |cos x| + c

1
Z Z Z
1  
8. dx = 1 + tan2 x dx = sec2 xdx = tan x + c
cos2 x
Z Z
1  
9. dx = 1 + cot2 x dx =  cot x + c
sin2 x
Z
1
10. dx = arctan x + c
1 + x2
Z
1
11. p dx = arcsin x + c
1  x2
Z
12. cosh xdx = sinh x + c
Z
13. sinh xdx = cosh x + c

Suppose we have two functions f (x) and g(x) with antiderivatives F (x) and G(x), respectively. Then we
know that
d
{F (x)  G(x)} = F 0 (x)  G0 (x) = f (x)  g(x)
dx
in other words, F  G is an antiderivative of f  g, which we can write as
Z Z Z
{f (x)  g(x)} dx = f (x)dx  g(x)dx.

Similarly,
d
(cF (x)) = cF 0 (x) = cf (x)
dx
implies that Z Z
cf (x)dx = c f (x)dx

if c is a constant.

Example 4 Evaluate the following indefinite integrals:


Z
1. (x2 + sin x)dx =?

Solution:
Z Z Z
(x2 + sin x)dx = x2 dx + sin xdx

x3
=  cos x + c
3

2
Z p4 7 1
2. (3 x3 + 5 + p )dx =?
x 6
x
Solution:
Z p Z
4 7 1 1
3
(3 x + 5 + p )dx = (3x3/4 + 7x5 + x1/2 )dx
x 6
x 6
4 7/4 7 4 1 1/2
= 3 x  x + 2x + c
7 4 6
12 7/4 7 4 1 1/2
= x  x + x +c
7 4 3
Z
4x10  2x4 + 15x2
3. ( )dx =?
x3
Solution:
Z Z  10 
4x10  2x4 + 15x2 4x 2x4 15x2
( )dx =  + dx
x3 x3 x3 x3
Z  
7 15
= 4x  2x + dx
x
1 8
= x  x2 + 15 ln |x| + c
2
Z  
1x
4. e + dx =?
x
Solution:
Z   Z Z
x 1 x 1
e + dx = e dx + dx
x x
= ex + ln |x| + c
Z
 
5. 3ex + 5 cos x  10 sec2 x dx =?

Solution:
Z Z Z Z
 x 
3e + 5 cos x  10 sec2 x dx = 3 ex dx + 5 cos xdx  10 sec2 xdx

= 3ex + 5 sin x  10 tan x + c


Z  
3
6. p + 6 sin x + 10 sinh x dx =?
1  x2
Solution:
Z   Z Z Z
3 1
p + 6 sin x + 10 sinh x dx = 3 p dx + 6 sin xdx + 10 sinh xdx
1  x2 1  x2
= 3 arcsin x  6 cos x + 10 cosh x + c

3
Z  
7  6 sin2 
7. d =?
sin2 
Solution:
Z   Z Z
7  6 sin2  1
d = 7 d  6 d
sin2  sin2 
= 7 cot   6 + c
Z
x2 + 1
8. p dx =?
x
Solution:
Z 2 Z Z
x +1
p dx = x dx + x1/2 dx
3/2
x
2 5/2
= x + 2x1/2 + c
5
Z
z 4  16
9. dz =?
z+2
Solution:
Z 4 Z 4 Z  2  
z  16 z  24 z  22 z 2 + 22
dz = dz = dz
z+2 z+2 z+2
Z   Z
(z  2) (z + 2) z 2 + 4  
= dz = (z  2) z 2 + 4 dz
z+2
Z
 3  1 2
= z + 4z  2z 2  8 dz = z 4 + 2z 2  z 3  8z + c
4 3
Z  
1
10. 2t + dt =?
1 + t2
Solution:
Z   Z Z
1 1
2t + dt = 2t dt + dt
1 + t2 1 + t2
2t
= + arctan t + c
ln 2

1.3 Substitution Method


Substitution method is the technique in integration that corresponds to the chain rule in di§erentiation. Let
y = F (u) be a function of u and let u = g(x) be a function of x. Then y can be considered as a function of
x by taking the composition of F with g:

y = F (g(x))

4
Suppose that the function g is di§erentiable on an open interval I and the function F is di§erentiable
on an open interval containing the image of I under g. Then by the Chain Rule, the composition function
F  g is di§erentiable on I and we have
d dy
F (g(x)) =
dx dx
dy du
= .
du dx
= F 0 (u).g 0 (x).

So, we get
d
F (u) = F 0 (g(x)).g 0 (x).
dx
This is the chain rule expressed in an alternative way. Since integration is the reverse process of di§er-
entiation, we have Z
F 0 (g(x)).g 0 (x)dx = F (g(x)) + c.

Denoting F 0 = f , the above integration formula becomes


Z
f (g(x)).g 0 (x)dx = F (g(x)) + c.

d 2
Example 5 Consider the function f (x) = 2xsin(x2 + 3). Notice that 2x = (x + 3). So let’s call
dx
g(x) = x2 + 3, and F (u) =  cos u, then
 
F (g(x)) =  cos x2 + 3

and
d
F (g(x)) = F 0 (g(x)).g 0 (x)
dx
= sin(x2 + 3).2x
so that Z
 
2xsin(x2 + 3)dx =  cos x2 + 3 + c.

The most transparent way of computing an integral by substitution is by following Leibniz and introduce
new variables. Thus to do the integral
Z
f (g(x))g 0 (x)dx

where f (u) = F 0 (u), we introduce the substitution u = g(x), and agree to write

g 0 (x)dx = du.
Then we get

5
Z Z
0
f (g(x))g (x)dx = f (u)du = F (u) + c

At the end of the integration we must remember that u really stands for G(x), so that
Z
f (g(x))g 0 (x)dx = F (g(x)) + c

Let’s do the previous example again. We want to find


Z
2xsin(x2 + 3)dx.

Let u = x2 + 3.Then
du = 2xdx and sin(x2 + 3) = sin u
so that Z Z
2xsin(x2 + 3)dx = sin udu =  cos u + c.

Finally we get rid of the subsitution variable u,and we get


Z
2xsin(x2 + 3)dx =  cos(x2 + 3) + c.
Z
x
Example 6 Let’s compute dx.
1 + x2
By using the substitution u = 1 + x2 , we have du = 2xdx and
Z Z
x 1 1 1
dx = du = ln |u| + c
1 + x2 2 u 2
Finally we get rid of the subsitution variable u,and we get
Z
x 1  
2
dx = ln 1 + x + c.
1 + x2 2
Example 7 Evaluate the following indefinite integrals by using substitution method:
Z
 4
1. 2x 1 + x2 dx = 15 x10 + x8 + 2x6 + 2x4 + x2 =?

Solution: By using the substitution u = 1 + x2 , we have du = 2xdx and


Z Z  5
 
2 4 4 u5 1 + x2
2x 1 + x dx = u du = +c= + c.
5 5
Z
xdx
2. p =?
x+1
Solution: By using the substitution u = x + 1, we have du = dx and
Z Z Z  
xdx u1 p 1
p dx = p du = u+ p du
x+1 u u

6
Z Z
p 1 2
= udu + p du = u3/2 + 2u1/2 + c
u 3
2 3/2 1/2
= (x + 1) + 2 (x + 1) + c.
3
Z
sin 2x
3. dx =?
1 + cos2 x
Solution: By using the substitution u = 1 + cos2 x, we have du = 2 cos x sin xdx =  sin 2xdx and
Z Z
sin 2x du  
2
dx = =  ln |u| + c =  ln 1 + cos2 x + c.
1 + cos x u
Z
2
4. (x + 1) ex +2x dx =?

Solution: By using the substitution u = x2 + 2x, we have du = (2x + 2) dx and


Z Z
x2 +2x 1 u 1 1 2
(x + 1) e dx = e du = eu + c = ex +2x + c.
2 2 2
Z
x3 + x 1
5. 11 dx =  10 =?
4 2
(x + 2x + 3) 40 (x + 2x2 + 3)
4
 
Solution: By using the substitution u = x4 + 2x2 + 3, we have du = 4x3 + 4x dx and
Z Z  
x3 + x 1 du 1 1 1
4 2 11 dx = 4u 11
=
4

10u 10
+c= 10 + c.
(x + 2x + 3) 40 (x + 2x2 + 3)
4

Z
xdx
6. p =?
3x  1
1
Solution: By using the substitution u = 3x  1, we have du = 3dx and x = (u + 1) ,so
3
Z Z 1 Z
xdx (u + 1) 1 1 u+1
p = 3 p du = p du
3x  1 u 3 9 u
Z  3/2 
1   1 u u1/2
= u1/2 + u1/2 du = + +c
9 9 3/2 1/2
q
2 3 2p
= (3x  1) + (3x  1) + c.
27 9
Z
dx
7. 2 =?
x (ln x)
dx
Solution: By using the substitution u = ln x, we have du = and
x
Z Z
dx du 1 1
2 = u 2
= +c=
u ln x
+ c.
x (ln x)

7
Z
8. tan x. ln(cos x)dx =?

Solution: By using the substitution u = cos x, we have du =  sin xdx and


Z Z Z
sin x ln u
tan x. ln(cos x)dx = . ln(cos x)dx =  du
cos x u
1
Again by using the substitution v = ln u, we have dv = du and
Z Z u
ln u 1 1 2 1 2
 du =  vdv =  v 2 + c =  (ln u) + c =  (cos x) + c.
u 2 2 2

1.3.1 Some Special Substitutions


1. If an indefinite integral has the integrand which contains an expression of the form
p
a2  x2
then we introduce the substitution x = a sin t. So we have

dx = a cos tdt

and p p
a2  x2 = a2  a2 sin2 t = |a| cos t.
2. If an indefinite integral has the integrand which contains an expression of the form
p
x2  a2
then we introduce the substitution x = a sec t. So we have

dx = a sec t tan tdt

and p p
x2  a2 = a2 sec2 t  a2 = |a| tan t.
3. If an indefinite integral has the integrand which contains an expression of the form
p
x2 + a2
then we introduce the substitution x = a tan t. So we have
a
dx = dt
cos2 t
and
p p |a|
x2 + a2 = a2 tan2 t + a2 = .
cos t
4. If an indefinite integral has the integrand which contains an expression of the form

ax2 + bx + c
then by using completing square method, expression can be written as

8
 2
p b b2
ax + p +c .
2 a 4a
p b
Then, the integral can be computed easier by making u = ax + p substitution.
2 a
Example 8 Evaluate the following indefinite integrals by using substitution method:
Z
xdx
1. p =?
4  x4
Solution: By using the substitution u = x2 .Then du = 2xdx and
Z Z du Z
xdx 2 1 du
p = p = p
4x 4 4u 2 2 2  u2
2

Again by using the substitution u = 2 sin t, we have du = 2 cos tdt and


p p
22  u2 = 22  22 sin2 t = 2 cos t. So
Z Z Z
1 du 1 2 cos tdt 1 t
p = = dt = + c
2 22  u2 2 2 cos t 2 2
u
Since u = 2 sin t, we have t = arcsin( ).Then
2
t 1 u 1 x2
+ c = arcsin( ) + c = arcsin( ) + c.
2 2 2 2 2
Z
dx
2. p =?
x x2 + 4
2

2
Solution: By using the substitution x = 2 tan t, we have dx = dt and
cos2 t
p p 2
x2 + 4 = 4 tan2 t + 4 = . So
cos t
Z Z 2 Z Z
dx 2t
dt 1 1 1 cos t
p = cos = dt = dt
x2 x2 + 1 2 4 1 4 sin2 t
4 tan2 t sin2 t
cos t cos t
Again by using the substitution u = sin t, we have du = cos tdt.So
Z Z
1 cos t 1 du 1 1
dt = = +c= +c
4 sin2 t 4 u2 4u 4 sin t
x
Since x = 2 tan t, by using t angled right triangle we have sin t = p
2
x +4
p
1 1 x2 + 4
 +c= + c.
4 sin t 4 x
Z
dx
3. p =?
x x2  9
Solution: By using the substitution x = 3 sec t. So we have dx = 3 sec t tan tdt and

9
p p
x2  32 = 9 sec2 t  9 = 3 tan t. Then
Z Z Z
dx 3 sec t tan tdt 1 1
p = = dt = t + c
2
x x 9 3 sec t.3 tan t 3 3
x
Since x = 3 sec t, we have t = arcsec
3
Therefore
Z x
dx 1 1
p = t + c = arcsec + c.
x x2  9 3 3 3
Z
dx
4. p =?
2x  x2
Solution: Firstly, let’s complete 2x  x2 to a square.
   
2x  x2 =  x2  2x =  (x  1)2  1 = 1  (x  1)2 .
Then
Z Z
dx dx
p = p
2x  x 2 1  (x  1)2
By using the substitution u = x  1, we have du = dx. So
Z Z Z
dx dx du
p = p = p = arcsin u + c = arcsin(x  1) + c.
2x  x 2 1  (x  1)2 1  u2
Z
dx
5. =?
4x2 + 4x + 2
Solution: Firstly, let’s complete 4x2 + 4x + 2 to a square.
4x2 + 4x + 2 = (2x + 1)2 + 1.
Then
Z Z
dx dx
=
4x2 + 4x + 2 (2x + 1)2 + 1
By using the substitution u = 2x + 1, we have du = 2dx. So
Z Z Z
dx dx 1 du 1 1
2
= 2
= 2
= arctan u + c = arctan (2x + 1) + c.
4x + 4x + 2 (2x + 1) + 1 2 u +1 2 2
Z
2x + 5
6. 2
dx =?
x + 4x + 13
Solution: Firstly, let’s complete x2 + 4x + 13 to a square.
x2 + 4x + 13 = (x + 2)2 + 9.
Then
Z Z Z
2x + 5 2x + 4 1
dx = dx + dx
x2 + 4x + 13 x2 + 4x + 13 (x + 2)2 + 9
Z Z
2x + 4 1 1
= dx +  x+2 2 dx
x2 + 4x + 13 9 + 1
| {z } | 3
{z }
I1
I2

10
To compute I1 , using the substitution u = x2 + 4x + 13, we have du = (2x + 4) dx. So
Z Z
2x + 4 du  
I1 = 2
dx = = ln |u| + c1 = ln x2 + 4x + 13 + c1
x + 4x + 13 u
x+2 dx
To compute I2 , using the substitution u = , we have du = . So
3 3
Z Z  
1 1 1 1 1 1 x+2
I2 =  x+2 2 dx = du = arctan u + c2 = arctan + c2
9 +1 9 u2 + 1 9 9 3
3
Therefore
Z  
2x + 5  2  1 x+2
 
dx = I1 + I2 = ln x + 4x + 13 + arctan + c.
x2 + 4x + 13 9 3

1.4 Integration of Rational Functions


P (x)
Recall that rational functions are functions that can be written in the form , where P (x) and Q(x) are
Q(x)
polynomials. Let deg(P (x)) = n and deg(Q(x)) = m

i) If n  m, then by long division, we can find a polynomial m(x) and a polynomial r(x) with degree less
than deg(Q(x)) such that

P (x) r(x)
= m(x) + , for all x with g(x) 6= 0.
Q(x) Q(x)
P (x)
Since polynomial functions can be integrated easily, to integrate , it su¢ces to know how to integrate
Q(x)
r(x)
.
Q(x)

P (x)
ii) If n < m,then by the help of partial fractions can be integrated easily.
Q(x)

a) If Q(x) = (x  x1 )(x  x2 )...(x  xm ),then

P (x) A1 A2 Am
= + + ... + .
Q(x) x  x1 x  x2 x  xm

b) If Q(x) = (x  x1 )k (x  x2 )...(x  xmk ),then

P (x) A1 A2 Ak Ak+1 Am
= + 2 + ... + k
+ + ... + .
Q(x) x  x1 (x  x1 ) (x  x1 ) x  x2 x  xmk

c) If Q(x) = (ax2 + bx + c)(x  x1 )...(x  xm2 ),then

P (x) A1 x + A2 A3 Am
= 2 + + ... + .
Q(x) ax + bx + c x  x1 x  xm2

11
Example 9 Evaluate the following indefinite integrals of rational functions by using substitution method:
Z
x
1. 2
dx =?
x  2x  3
Solution: Since x2  2x  3 = (x  3)(x + 1) the partial fraction decomposition of the given rational
function takes the form
x A B
2
= + .
x  2x  3 x3 x+1
Then we get
x = A(x + 1) + B(x  3).
Putting x = 1 and x = 3, we get 1 = 4B and 3 = 4A, respectively.
3 1
Thus, we obtain A = and B = .
4 4
Therefore we have partial fraction decomposition
x 3 1 1 1
= + .
x2  2x  3 4x3 4x+1
So
Z Z Z
x 3 1 1 1
2
dx = dx + dx
x  2x  3 4 x3 4 x+1
3 1
= ln |x  3| + ln |x + 1| + c.
4 4
Z
2x + 3
2. dx =?
x2  2x + 1
Solution: Since x2  2x + 1 = (x  1)2 the partial fraction decomposition of the given rational function
takes the form
2x + 3 A B
= + .
x2  2x + 1 x  1 (x  1)2
Then we get
2x + 3 = A(x  1) + B.
Putting x = 1 and x = 0, we get 5 = B and 3 = A + B respectively, which yields A = 2
Therefore we have partial fraction decomposition
2x + 3 2 5
= + .
x2  2x + 1 x  1 (x  1)2
So
Z Z Z
2x + 3 2 5
dx = dx + 2 dx
x2  2x + 1 x1
| {z } | (x {z
1)
}
I1 I2
Z
2
I1 = dx = 2 ln |x  1| + c1
x1
To compute I2 , using the substitution u = x  1, we have du = dx. So

12
Z Z
5 du 5 5
I2 = 2 dx = 5 2
=  + c2 =  + c2 .
(x  1) u u x1
Then, we obtain
Z
2x + 3 5
2
dx = I1 + I2 = 2 ln |x  1|  + c.
x  2x + 1 x1
Z
2x + 3
3. dx =?
x4  2x3 + 2x2  2x + 1
Solution: Since x4  2x3 + 2x2  2x + 1 = (x  1)2 (x2 + 1) the partial fraction decomposition of the
given rational function takes the form
2x + 3 A B Cx + D
= + + 2 .
x4  2x3 + 2x2  2x + 1 x  1 (x  1)2 x +1
Then we get
2
2x + 3 = A (x  1) (x2 + 1) + B(x2 + 1) + (Cx + D) (x  1)
This implies that
2x + 3 = x3 (A + C) + x2 (A + B  2C + D) + x(A + C  2D)  A + B + D.
So, we get
A + C = 0, A + B  2C + D = 0, A + C  2D = 2, and A + B + D = 3.
3 5 3
Then, A = , B = , C = , and D = 1.
2 2 2
Therefore we have partial fraction decomposition
3 5 3
2x + 3 x1
= 2 + 2 + 2 .
x4  2x3 + 2x2  2x + 1 x  1 (x  1)2 x2 + 1
So
Z Z Z Z Z
2x + 3 3 dx 5 dx 3 2x dx
4 3 2
dx = + 2 +
2
dx  2
x  2x + 2x  2x + 1 2 x1 2 (x  1) 4 x +1 x +1
3 5 1 3  
= ln |x  1|  + ln x2 + 1  arctan x + c.
2 2x1 4
Z
x3 + 2
4. dx =?
x3  x
Z 3 Z 3 Z   Z
x +2 x x+x+2 x+2 x+2
Solution: dx = dx = 1+ 3 dx = x + dx
x3  x x3  x x x x3  x
x+2
Since x3  x = x(x  1)(x + 1) the partial fraction decomposition of 3 takes the form
x x
x+2 A B C
3
= + + .
x x x x1 x+1
Then we get
x + 2 = A(x2  1) + B(x2 + x) + C(x2  x)
This implies that

13
x + 2 = x2 (A + B + C) + x(B  C)  A.
3 1
It follows that A = 2, B = and C = .
2 2
Therefore we have partial fraction decomposition
x+2 2 3 1 1 1
3
= + + .
x x x 2x1 2x+1
So
Z 3 Z
x +2 x+2
dx = x + dx
x3  x x3  x
Z Z Z
dx 3 dx 1 dx
=x2 + +
x 2 x1 2 x+1
3 1
= x  2 ln |x| + ln |x  1| + ln |x + 1| + c.
2 2
Z 3 2
x + 7x + 16x + 11
5. dx =?
x2 + 4x + 3
 
Solution: Since x3 + 7x2 + 16x + 11 = x2 + 4x + 3 (x + 3) + x + 2, we can write
x3 + 7x2 + 16x + 11 x+2
=x+3+ 2 .
x2 + 4x + 3 x + 4x + 3
Then, we have
Z 3 Z  
x + 7x2 + 16x + 11 x+2
dx = x+3+ 2 dx
x2 + 4x + 3 x + 4x + 3
Z Z Z
x+2
= xdx + 3 dx + 2
dx
x + 4x + 3
Z
x2 x+2
= + 3x + dx.
2 x2 + 4x + 3

By using the substitution u = x2 + 4x + 3, we have du = (2x + 4) dx. Then we obtain


Z 3 Z
x + 7x2 + 16x + 11 x2 1 du
dx = + 3x +
x2 + 4x + 3 2 2 u
x2 1
= + 3x + ln |u| + c
2 2
x2 1  2 
= + 3x + ln x + 4x + 3 + c.
2 2

1.5 Integration by Parts


The technique in integration that corresponds to the product rule in di§erentiation is called integration by
parts. Let f and g be functions that are di§erentiable on an open interval (a, b). By the product rule, we
have
d
(f (x)g(x)) = f 0 (x)g(x) + f (x)g 0 (x), a<x<b
dx

14
which written in terms of integration, becomes
Z Z
f (x)g(x)dx + f (x)g 0 (x)dx = f (x)g(x),
0
a < x < b.

If one of the two integrals on the left side is easy to find, then we can find the other one. By symmetry, we
may assume that the first integral is easy to find, then in this case, we can find the second integral by the
following:
Z Z
f (x)g 0 (x)dx = f (x)g(x)  f 0 (x)g(x)dx.

In the above equation, by putting u = f (x) and v = g(x) so that du = f 0 (x)dx and dv = g 0 (x)dx, we get
Z Z
udv = uv  vdu.

Here, we have to choose suitable u and dv. From the chosen dv, we have to find v. This is done by
integration.
A Guide for Integration by Parts: Treat the integrand as a product of two functions. Choose u to
be one of the two functions such that

• the other function can be integrated easily. Choose dv = (the other f unction)dx;
Z Z
• the new integral vdu is easier to find than the original integral udv.
Z
Example 10 Find x cos xdx.

The integrand is a product of two functions. There are two options for u and dv.

x2
• Put u = cos x and dv = xdx. Then we have du =  sin xdx and we can take v = . Note that
Z Z 2
2
x
vdu =  sin xdx which is even more complicated then the original integral.
2
Z
• Put u = x and dv = cos xdx.Then we have du = dx and we can take v = sin x. Note that vdu =
Z
sin xdx is easy to find.(the method works). From these we get

Z Z
x cos xdx = udv
Z
= uv  vdu
Z
= x sin x  sin xdx

= x sin x + cos x + c

15
Example 11 Evaluate the following indefinite integrals by using integration by parts method:
Z
1. x2 ln xdx =?

1 x3
Solution: Put u = ln x and dv = x2 dx.Then we have du = dx and we can take v = . From these
x 3
we get
Z Z
x2 ln xdx = udv
Z
= uv  vdu
Z
x3 x3 1
= ln x  dx
3 3 x
Z
x3 x2
= ln x  dx
3 3
x3 x3
= ln x  + c.
3 9
Z
2. x3 ex dx =?

Solution: Put u = x3 and dv = ex dx.Then we have du = 3x2 dx and we can take v = ex . From these
we get
Z Z
x3 ex dx = udv
Z
= uv  vdu
Z
= x3 ex  3 x2 ex dx
Z
Here, to find x2 ex dx,we will apply integration by parts method again. Put u = x2 and dv =
ex dx.Then we have du = 2xdx and we can take v = ex . From these we get
Z Z
x3 ex dx = x3 ex  3 x2 ex dx
Z
= x3 ex  3 udv
 Z 
3 x
= x e  3 uv  vdu
 Z 
= x3 ex  3 x2 ex  2 xex dx

16
Z
Here also, to find xex dx,we will apply integration by parts method again. Put u = x and dv = ex dx.
Then we have du = dx and we can take v = ex . From these we get
Z  Z 
x3 ex dx = x3 ex  3 x2 ex  2 xex dx
Z
= x3 ex  3x2 ex  6 udv
 Z 
3 x 2 x
= x e  3x e  6 uv  vdu
 Z 
= x3 ex  3x2 ex  6 xex  ex dx

= x3 ex  3x2 ex  6xex  6ex + c


 
= x3  3x2  6x  6 ex + c.
Z
3. e2x cos 3xdx =?

e2x
Solution: Put u = cos 3x and dv = e2x dx.Then we have du = 3 sin 3xdx and we can take v = .
2
From these we get
Z Z
e2x cos 3xdx = udv
Z
= uv  vdu
Z
e2x 3
= cos 3x. + 2 sin 3x.e2x dx
2
Z
Here, to find sin 3x.e2x dx,we will apply integration by parts method again. Put u = sin 3x and
e2x
dv = e2x dx. Then we have du = 3 cos 3xdx and we can take v = . From these we get
2

17
Z Z
e2x 3
e2x cos 3xdx = cos 3x. + sin 3x.e2x dx
2 2
Z
e2x 3
= cos 3x. + udv
2 2
 Z 
e2x 3
= cos 3x. + uv  vdu
2 2
2x
 Z 
e 3 e2x 3 2x
= cos 3x. + sin 3x.  e cos 3xdx
2 2 2 2
Z
e2x 3 2x 9
= cos 3x. + sin 3x.e  e2x cos 3xdx.
2 4 4

Then, we have
Z
13 1 2x 3
e2x cos 3xdx = e cos 3x + e2x sin 3x
4 2 4
Z  
4 1 2x 3
=) e2x cos 3xdx = e cos 3x + e2x sin 3x + c.
13 2 4

Z
4. x2 sin xdx =?

Solution: Put u = x2 and dv = sin xdx.Then we have du = 2xdx and we can take v =  cos x. From
these we get
Z Z
x2 sin xdx = udv
Z
= uv  vdu
Z
2
= x cos x + 2 x cos xdx

= x2 cos x + 2 (x sin x + cos x) + c


Z
5. sin x ln(cos x)dx =?

 sin x
Solution: Put u = ln(cos x) and dv = sin xdx.Then we have du = dx =  tan xdx and we can
cos x
take v =  cos x. From these we get

18
Z Z
sin x ln(cos x)dx = udv
Z
= uv  vdu
Z
=  cos x ln(cos x)  cos x tan xdx
Z
=  cos x ln(cos x)  sin xdx

=  cos x ln(cos x) + cos x + c.


Z
6. arcsin xdx =?

1
Solution: Put u = arcsin x and dv = dx.Then we have du = p dx and we can take v = x. From
1  x2
these we get
Z Z
arcsin xdx = udv
Z
= uv  vdu
Z
x
= x arcsin x  p dx
1  x2
| {z }
I1

To compute the integral I1 , using the substitution w = 1  x2 , we have dw = 2x. Then we have
Z  Z
1 dw
arcsin xdx = x arcsin x   p
2 w
1 p
= x arcsin x + 2 w + c
2
p
= x arcsin x + 1  x2 + c.
Z
p
7. 1  x2 dx =?
p x
Solution: Put u = 1  x2 and dv = dx.Then we have du = p dx and we can take v = x. From
1  x2
these we get

19
Z Z
p
I= 1  x2 dx = udv
Z
= uv  vdu
Z
p x2
= x 1  x2  p dx
1  x2
| {z }
I1

Z Z
x2 1  x2  1
I1 = p dx = p dx
1  x2 1  x2
Z Z
1  x2 dx
= p dx  p
1x 2 1  x2
Z
p
= 1  x2 dx  arcsin x

= I  arcsin x + c1
Therefore we obtain
p p
I = x 1  x2  I1 = x 1  x2  I + arcsin x + c1
p
=) 2I = x 1  x2 + arcsin x + c1
p
x 1  x2 + arcsin x
=) I = + c.
2
Z
p
8. ln(x + 1 + x2 )dx =?
x
p 1+ p
2 1
Solution: Put u = ln(x + 1 + x2 ) and dv = dx.Then we have du = p1 + x dx = p dx
x + 1 + x2 1 + x2
and we can take v = x. From these we get
Z Z
p
ln(x + 1 + x2 )dx = udv
Z
= uv  vdu
Z
p x
= x ln(x + 1 + x2 )  p dx
1 + x2
| {z }
I1

To compute I1 , using substitution w = 1 + x2 , we have 2xdx = du. Then we get

20
Z Z
p p 1 dw
ln(x + 1 + x2 )dx = x ln(x + 1 + x2 ) 
p
2 w
p 1 p
= x ln(x + 1 + x2 )  2 w + c
2
p p
= x ln(x + 1 + x )  1 + x2 + c.
2

Z
arcsin x
9. p dx =?
1+x
dx dx
Solution: Put u = arcsin x and dv = p .Then we have du = p and we can take v =
p 1 + x 1  x2
2 1 + x. From these we get
Z Z
arcsin x
p dx = udv
1+x
Z
= uv  vdu
Z p
p 1+x
= 2 1 + x arcsin x  2 p dx
1  x2
Z
p 1
= 2 1 + x arcsin x  2 p dx
1x
p p
= 2 1 + x arcsin x + 4 1  x + c.

1.6 Integration of Trigonometric Functions


i) Integrals involving sin ax. cos bx, sin ax. sin bx, cos ax. cos bx can be computed by using the following ad-
dition formulas for sine and cosine functions
sin(a + b)x = sin ax. cos bx + cos ax. sin bx
sin(a  b)x = sin ax. cos bx  cos ax. sin bx
cos(a + b)x = cos ax. cos bx  sin ax. sin bx
cos(a  b)x = cos ax. cos bx + sin ax. sin bx

Z
Example 12 Find sin 3x. cos 4xdx.
By using addition formulas for sine function, we get

sin(4 + 3)x = sin 4x. cos 3x + cos 4x. sin 3x (1)

and

sin(4  3)x = sin 4x. cos 3x  cos 4x. sin 3x. (2)
By taking (1)  (2) ,we have

21
1
2 sin 3x. cos 4x = sin 7x  sin x =) sin 3x. cos 4x = (sin 7x  sin x)
2
Z Z
1
=) sin 3x. cos 4xdx = (sin 7x  sin x) dx
2
1 1
=  cos 7x + cos x + c
14 2
Z
Example 13 Find cos 5x. cos 3xdx.
By using addition formulas for cosine function, we get

cos(5 + 3)x = cos 5x. cos 3x  sin 5x. sin 3x (3)


and

cos(5  3)x = cos 5x. cos 3x + sin 5x. sin 3x (4)


By taking (3) + (4) ,we have
1
2 cos 5x. cos 3x = cos 8x + cos 2x =) cos 5x. cos 3x = (cos 8x + cos 2x)
2
Z Z
1
=) cos 5x. cos 3xdx = (cos 8x + cos 2x) dx
2
1 1
= sin 8x + sin 2x + c.
16 4
Z
Example 14 Find sin 4x. sin 2xdx.
By using addition formulas for cosine function, we get

cos(4  2)x = cos 4x. cos 2x + sin 4x. sin 2x (5)


and

cos(4 + 2)x = cos 4x. cos 2x  sin 4x. sin 2x (6)


By taking (5)  (6) ,we have
1
2 sin 4x. sin 2x = cos 2x  cos 6x =) sin 4x. sin 2x = (cos 2x  cos 6x)
2
Z Z
1
=) sin 4x. sin 2xdx = (cos 2x  cos 6x) dx
2
1 1
= sin 2x  sin 6x + c.
4 12

22
ii) Consider the integrals of the form
Z
sinm x. cosn xdx.

If m is an odd positive integer, the integral can be done easily by substitution. Say m = 2k + 1, where k is
an integer, then by using the identity sin2 x + cos2 x = 1, we can rewrite the integral in the form
Z
 k
sin x 1  cos2 x cosn xdx

which can be integrated using the substitution u = cos x


Z
Example 15 Find sin5 x. cos4 xdx.

By using the identity sin2 x + cos2 x = 1,we can write


Z Z
 2
sin5 x. cos4 xdx = sin x 1  cos2 x cos4 xdx.

Then using the substitution u = cos x and so du =  sin xdx, we get


Z Z
5
 2
sin x. cos xdx = sin x 1  cos2 x cos4 xdx
4

Z
=  (1  u2 )2 u4 du
Z
 8 
= u  2u6 + u4 du

u9 u7 u5
= +2  +c
9 7 5
cos9 x cos7 x cos5 x
= +2  + c.
9 7 5

iii) Consider the integrals of the form


Z
sinm x. cosn xdx.

If n is an odd positive integer, the integral can be done easily by substitution. Say n = 2k + 1, where k is
an integer, then by using the identity sin2 x + cos2 x = 1, we can rewrite the integral in the form
Z
 k
sinm x 1  sin2 x cos xdx

which can be integrated using the substitution u = sin x.


Z
Example 16 Find sin4 x. cos3 xdx.

23
By using the identity sin2 x + cos2 x = 1,we can write
Z Z
 
sin x. cos xdx = sin4 x 1  sin2 x cos xdx.
4 3

Then using the substitution u = sin x and so du = cos xdx, we get


Z Z
 
sin4 x. cos3 xdx = sin4 x 1  sin2 x cos xdx
Z
= u4 (1  u2 )du
Z
 
= u6 + u4 du

u7 u5
= + +c
7 5
sin7 x sin5 x
= + + c.
7 5
Z
Example 17 Find cos5 axdx.

2 2
By
Z using the identity Zsin ax + cos ax = 1,we can write
cos5 axdx = cos4 ax cos axdx
Z .
 2
= 1  sin2 ax cos axdx

Then using the substitution u = sin ax and so du = a cos axdx, we get


Z Z
 2
cos5 axdx = 1  sin2 ax cos axdx
Z
1
= (1  u2 )2 du
a
Z
1  
= u4  2u2 + 1 du
a
 
1 u5 u3
= 2 +u +c
a 5 3 
5
1 sin ax sin3 ax
= 2 + sin ax + c.
a 5 3

iv) Consider the integrals of the form


Z
sinm x. cosn xdx.

If m and n are both even positive integer, then we can use the doubleangle formulas

24
1 + cos 2x 1  cos 2x
cos2 x = and sin2 x = .
2 2
Z
Example 18 Find sin2 x. cos2 xdx.

By using the doubleangle formulas

1 + cos 2x 1  cos 2x
cos2 x = and sin2 x =
2 2
we can write
Z Z   
2 2 1  cos 2x 1 + cos 2x
sin x. cos xdx = dx
2 2
Z
1  
= 1  cos2 2x dx
4
Z   
1 1 + cos 4x
= 1 dx
4 2
Z
1
= (1  cos 4x) dx
8
 
1 sin 4x
= x + c.
8 4
x
v) If the integral involve sin x or cos x functions, then using the substitution t = tan makes the
2
computation easier. By this way, we have
2
dx = dt.
1 + t2
x
Also by using angled right triangle we have
2
x x
sin x = 2 sin cos x
2 2 cos x = 2 cos2 1
2
t 1
= 2p p and 1  t2
1+t 2 1 + t2 = .
1 + t2
2t
= .
1 + t2
Z
dx
Example 19 Find .
cos x + 2 sin x
x 2 1  t2 2t
By using the substitution t = tan ,we have dx = 2
dt, cos x = and sin x = So, we get
2 1+t 1 + t2 1 + t2

25
Z 2dt Z
dx 1 + t2
=
cos x + 2 sin x 1  t2 2.2t
+
1 + t2 1 + t2
Z
dt
=2
1 + 4t  t2
Z
dt
=2 2 .
 (t  2) + 5
Z
2 dt
=  2
5 t2
 p +1
5
t2 p
Let’s say p = u, then 5du = dt. We have
Z 5 p Z
dx 2 5 du
=
cos x + 2 sin x 5 u2 + 1
p
2 5
= arctan u
5
p  
2 5 t2
= arctan p
5 5
p  
2 5 tan x  4
= arctan p .
5 2 5

Problems
Evaluate the following indefinite integrals.
Z
1. 3 sec2 xdx

Z  2
1
2. 1+ dx
x
Z
3. 2x(x2 + 1)9 dx
Z
p
4. x4 x5 + 6dx
Z
3
5. x2 ex 1
dx
Z
x3 + x
6. 11 dx
(x4 + 2x2 + 3)

26
Z   1
x2  1 ex+ x
7. dx
x2
Z
sin x1
8. dx
x2
Z
sin (ln x)
9. dx
x
Z
1
10. dx
x ln x
Z
4
11. sin(1  x) (2  cos(1  x)) dx
Z
12. sin3 x cos8 xdx
Z
13. sin4 xdx
Z
14. sin4 x cos5 xdx
Z
15. cos x sin4 (sin x) dx
Z p
x
16. xe dx
Z
x3 + 3x2
17. dx
x2 + 1
Z
x
18. dx
2x  1
Z
x+4
19. dx
x2  5x + 6
Z
x2 + 3x + 2
20. dx
x (x2 + 1)
Z
1
21. dx
x3 +1
Z
x2 + 2
22. dx
4x5 + 4x3 + x
Z
1
23. 3/2
dx
(5  x2 )

27
Z
1
24. p dx
4 + x2
Z
1
25. p dx, where a > 0.
x2  a2
Z
1
26. p dx
2x  x2
Z
1
27. p dx
1+ 2x
Z
1
28. d
2 + cos 

28

You might also like